+7 495 120-13-73 | 8 800 500-97-74

(для регионов бесплатно)

Содержание

§50. Активное сопротивление в цепи переменного тока

Ток и напряжение. При включении в цепь переменного тока активного сопротивления R (рис. 175, а) напряжение и источника создает в цепи ток i. Если напряжение и изменяется по синусоидальному закону u = Uт sin ?t, то ток i также изменяется синусоидально:

i = Iт sin ?t

При этом

Iт = Uт / R

Таким образом, ток и напряжение изменяются по одному и тому же закону; они одновременно достигают своих максимальных значений и одновременно проходят через нуль (рис. 175,б). Следовательно, при включении в цепь переменного тока активного сопротивления ток и напряжение совпадают по фазе (рис. 175, в).

Если обе части равенства Iт = Uт / R разделить на ?2, то получим выражение закона Ома для рассматриваемой цепи для действующих значений напряжения и тока:

I = U / R

Следовательно, для цепи переменного тока, содержащей только активное сопротивление, этот закон имеет такую же математическую форму, как и для цепи постоянного тока.

Электрическая мощность. Электрическая мощность р в цепи с активным сопротивлением в любой момент времени равна произведению мгновенных значений силы тока i и напряжения и. Следовательно, мгновенная мощность р не является постоянной величиной, как при постоянном токе, а изменяется по кривой (см. рис. 175,б). Эту кривую можно также получить графически, перемножая ординаты кривых силы тока i и напряжения и при различных углах ?t. Изменение мощности происходит с двойной частотой ?t по отношению к изменению тока и напряжения, т. е. один период изменения мощности соответствует половине периода изменения тока и напряжения. Все значения мощности являются положительными. Физически положительное значение мощности означает, что энергия передается от источника электрической энергии к приемнику. Максимальное значение мощности при ?t = 90° и ?t = 270°

Pmax = UтIт = 2UI

Рис. 175. Схема включения в цепь переменного тока активного сопротивления (а), кривые тока i, напряжения и, мощности р (б) и векторная диаграмма (в)

Практически об энергии W, создаваемой электрическим током, судят не по максимальной мощности, а по средней мощности Рср = Р, так как эта энергия может быть выражена как произведение среднего значения мощности Р на время протекания тока:

W = Pt.

Кривая мгновенной мощности симметрична относительно линии АБ, которая соответствует среднему значению мощности Р. Поэтому

P = Pmax / 2 = UI

Используя формулу (67) закона Ома, активную мощность можно выразить также в виде P = I2R или P=U2/R.

В электротехнике среднюю мощность, потребляемую активным сопротивлением, обычно называют активной мощностью, или просто мощностью, и обозначают буквой Р.

Поверхностный эффект. Следует отметить, что активное сопротивление проводников в цепи переменного тока всегда больше их сопротивления в цепи постоянного тока. Переменный ток i не протекает равномерно по всему поперечному сечению проводника, как постоянный ток i, а вытесняется на его поверхность (рис. 176, а). Поэтому полезное сечение проводника как бы уменьшается и сопротивление его при переменном токе возрастает. Это явление носит название поверхностного эффекта. Неравномерное распределение переменного тока по поперечному сечению проводника объясняется действием э. д. с. самоиндукции, индуцированной в проводнике магнитным полем, которое создается проходящим по проводнику током I. Это магнитное поле действует не только в пространстве, окружающем проводник (внешний поток Ф

2), но и внутри самого проводника (внутренний поток Ф
2
) (рис. 176,б). Поэтому слои проводника, расположенные ближе к его центру, будут охватываться большим магнитным потоком, чем слои, расположенные ближе к его поверхности, и э. д. с. самоиндукции, индуцированная во внутренних слоях, будет большей, чем во внешних. Поскольку э. д. с. самоиндукции препятствует изменению

Рис. 176. Схема протекания постоянного I и переменного i токов по проводнику (а) и возникновение поверхностного эффекта (б)

Рис. 177. Схема термообработки деталей токами высокой частоты: 1 — высокочастотный индуктор; 2 — закаливаемая деталь; 3 — разогретый слой

тока, последний будет стремиться пройти там, где э. д. с. самоиндукции имеет наименьшее значение, т. е. пройдет преимущественно по поверхностным слоям проводника. В результате этого плотность тока У в поверхностных слоях будет больше, чем во внутренних. Чем больше частота тока, тем больше э. д. с. самоиндукции индуцируется во внутренних слоях проводника и тем в большей степени ток вытесняется на поверхность.

При частоте 50 Гц увеличение сопротивления медных и алюминиевых проводников при малом их диаметре практически ничтожно, и сопротивление таких проводников в цепях переменного и постоянного тока можно считать одинаковым. Но для медных и алюминиевых проводников диаметром свыше 10 мм, а для стальных проводников при еще меньших диаметрах необходимо при расчетах учитывать влияние поверхностного эффекта на их активное сопротивление.

При токах высокой частоты, принятых в радиотехнике, телевидении и различных высокочастотных установках, с целью лучшего использования металла проводников их обычно изготовляют полыми.

На свойстве переменного тока высокой частоты протекать, главным образом, по поверхности проводников основаны различные методы высокочастотной закалки и термообработки. Например, при высокочастотной термообработке деталей вихревыми токами (рис. 177) эти токи индуцируются в основном в поверхностном слое металла. Они быстро разогревают поверхностные слои обрабатываемой детали, раньше, чем ее внутренняя часть успеет заметно нагреться за счет теплопроводности металла.

Закон Ома для переменного тока

 

 Мы с вами знаем формулировку закона Ома для цепей постоянного тока, которая гласит, что ток в такой цепи прямо пропорционален напряжению на элементе цепи и обратно пропорционален сопротивлению этого элемента постоянному току, протекающему через него.

Однако при изучении цепей переменного тока стало известно, что оказывается кроме элементов цепей с активным сопротивлением, есть элементы цепи с так называемым реактивным сопротивлением, то есть индуктивности и емкости (катушки и конденсаторы).

В цепи, содержащей только активное сопротивление, фаза тока всегда совпадает с фазой напряжения (рис 1.), т. е. сдвиг фаз тока и напряжения в цепи с чисто активным сопротивлением равен нулю.

Рисунок 1. Напряжение и ток в цепи с чисто активным сопротивлением. Сдвиг фаз между током и напряжение в цепи переменного тока с чисто активным сопротивлением всегда равен нулю

Отсюда следует, что угол между радиус-векторами тока и напряжения также равен нулю.

Тогда, падение напряжения на активном сопротивлении определяется по формуле:

  (1)

где, U-напряжение на элементе цепи,

I – ток через элемент цепи

R – активное сопротивление элемента

Формула (1) применима как для амплитудных, так и для эффективных значений тока и напряжения:

 (2)

где, Um-амплитудное значение напряжения на элементе цепи,

Im – амплитудное значение тока через элемент цепи

R – активное сопротивление элемента

В цепи, содержащей чисто реактивное сопротивление — индуктивное или емкостное, — фазы тока и напряжения сдвинуты друг относительно друга на четверть периода, причем в чисто индуктивной цепи фаза тока отстает от фазы напряжения (рис. 2), а в чисто емкостной цепи фаза тока опережает фазу напряжения (рис. 3).

Рисунок 2. Напряжение и ток в цепи с чисто индуктивным сопротивлением. Фаза тока отстает от фазы напряжения на 90 градусов.

 

Рисунок 3. Напряжение и ток в цепи с чисто емкостным сопротивлением. Фаза тока опережает фазу напряжения на угол 90 градусов.

Отсюда следует, что в чисто реактивной цепи угол между радиус-векторами тока и напряжения всегда равен 90°, причем в чисто индуктивной цепи радиус-вектор тока при вращении движется позади радиус-вектора напряжения, а в чисто емкостной цепи он движется впереди радиус-вектора напряжения.

Падения напряжения на индуктивном и емкостном сопротивлениях определяются соответственно по формулам:

 
 (3)
 

 (4)

где — UL-падение напряжение на чисто индуктивном сопротивлении ;

UС—падение напряжения на чисто емкостном сопротивлении;

I— значение тока в через реактивное сопротивление;

L— индуктивность реактивного элемента;

C— емкость реактивного элемента;

ω— циклическая частота.

Эти формулы применимы как для амплитудных, так и для эффективных значений тока и напряжения синусоидальной формы. Однако здесь следует отметить, что они ни в коем случае не применимы для мгновенных значений тока и напряжения, а также и для несинусоидальных токов.

Приведенные выше формулы являются частными случаями

закона Ома для переменного тока.

Следовательно, полный закон Ома для переменного тока будет иметь вид:

(5)

Где Z – полное сопротивление цепи переменного тока.

Теперь остается только вычистислить полное сопротивление цепи, а оно зависит непосредсвенно от какие активные и реактивные элементы присутсвуют в цепи и как они соединены.

Закон Ома для различных типовых цепей переменного тока

Давайте выясним, как будет выглядеть закон Ома для цепи переменного тока, состоящей из активного и индуктивного сопротивлений, соединенных последовательно (рис. 4.)

Рисунок 4. Цепь переменного тока с последовательным соединением активного и индуктивного сопротивления.

Закон Ома для переменного синусоидального тока в случае последовательного соединения активного и индуктивного сопротивлений выражается следующей формулой:

 

(6)

где —эффективное значение силы тока в А;

U—эффективное значение напряжения в В;

R—активное сопротивление в Ом;

ωL—индуктивное сопротивление в ом.

Формула (6) будет также действительной, если в нее подставить амплитудные значения тока и напряжения.

В цепи, изображенной на рис. 5, соединены последовательно активное и емкостное сопротивления.

Рисунок 5. Цепь переменного тока с последовательным соединением активного и емкосного сопротивления.

А закон Ома для такой цепи принимает вид:

(7)

В общем случае, когда цепь содержит все три вида сопротивлений (рис. 6),

Рисунок 6. Цепь переменного тока с последовательным соединением активного, индуктивного и емкосного сопротивления.

Закон Ома при последовательном соединении активного, индуктивного и емкостного сопротивлений будет выглядеть так:

(8)

где I-сила тока в А;

U-напряжение в В;

R-активное сопротивление в Ом;

ωL-индуктивное сопротивление в Ом;

1/ωС-емкостное сопротивление в Ом.

Формула (8) верна только для эффективных и амплитудных значений синусоидального тока и напряжения.

Для того, что бы определить ток в цепях с параллельным соединением элементов (рисунок 7), то необходимо так же вычислить полное сопротивление цепи, как это делать можно прсмотреть здесь, зтем подставить значение полного сопротивления в общую формулу для закона Ома (5).

Рисунок 7. Полное сопротивление цепи при параллельном соединении активного и реактивных элементов. а) — параллельное соединение R и L; б) — параллельное соединение R и C.

 

Тоже самое касается и вычисления тока в колебательном контуре изображенном на рисунке 8.

Рисунок 8. Эквивалентная схема колебательного контура.

 

Таким образом закон Ома для переменного тока можно сформулировать следующим образом.

Значение тока в цепи переменного тока прямо пропорционально напряжению в цепи (или на участке цепи) и обратно пропорционально полному сопротивлению цепи (участка цепи)

 

ПОНРАВИЛАСЬ СТАТЬЯ? ПОДЕЛИСЬ С ДРУЗЬЯМИ В СОЦИАЛЬНЫХ СЕТЯХ!

Похожие материалы:

Добавить комментарий

Цепь переменного тока с активным сопротивлением

Когда в электрическую цепь переменного тока подключается активное сопротивление R, то под воздействием разницы потенциалов источника в цепи начинает течь ток I. В тех случаях, когда изменение напряжения происходит по синусоидальному закону, который выражается, как u = Um sin ωt, то изменение тока i также идет по синусоиде:

Активное сопротивление

 

 

i = Im sin ωt

При этом

Так что получается, что изменение напряжения и тока происходят по одинаковым законам. При этом через нулевое значение они проходят одновременно и своих максимальных значений также достигают одновременно. Из этого следует, что когда в электрическую цепь переменного тока подключается активное сопротивление R, то напряжение и ток совпадают по фазе.

Мощность, ток, напряжение

Если взять равенство Im = Um / R и каждую из его частей разделить на √2, то в итоге получится ни что иное, как закон Ома, применимый для той цепи, которая рассматривается: I = U/R.

Таким образом, получается, что это основополагающий закон для той цепи, которая имеет в своем составе только активное сопротивление, с точки зрения математики имеет такую же форму, что и для цепи тока постоянного.

Электрическая мощность

Такой показатель, как электрическая мощность P для цепи, имеющей в своем составе активное сопротивление, равняется произведению мгновенного значения напряжения U на мгновенное значение силы тока i в любой момент времени. Из этого следует, что в цепях переменного тока, в отличие от цепей тока постоянного, мгновенная мощность P – величина непостоянная, а ее изменение происходит по кривой. Для того чтобы получить ее графическое представление, необходимо ординаты кривых напряжения U и силы тока i перемножить при разных углах ωt. Мощность изменяется по отношению к изменению тока с двойной частотой ωt. Это означает, что половине периода изменения напряжения и тока соответствует один период изменения мощности. Следует заметить, что абсолютно все значения, которые может принимать мощность, являются положительными величинами. С точки зрения физики это означает, что от источника к приемнику передается энергия. Своих максимальных значений мощность достигает тогда, когда ωt = 270° и ωt = 90°.

В практическом отношении о той энергии W, которую создает электрический ток, принято судить по средней мощности, выражаемой формулой Рср = Р, а не по мощности максимальной. Ее можно определить, перемножив на время протекания тока среднее значение мощности W = Pt.

Относительно линии АБ, соответствующей среднему значению мощности P, кривая мгновенной мощности симметрична. По этой причине

P = Pmax / 2 = UI

Если использовать закон Ома, то можно выразить активную мощность в следующем виде:

P = I2R или P = U2/R.

Специалисты в области электротехники ту среднюю мощность, которую потребляет активное сопротивление, чаще всего именуют или просто мощностью, или активной мощностью, а для ее обозначения используется буква P.

Поверхностный эффект

Необходимо особо отметить такую особенность проводников, включенных в сеть переменного тока: их активное сопротивление во всех случаях оказывается больше, чем если бы они были включены в сеть тока постоянного. Причина этого состоит в том, что переменный ток не протекает равномерно распределяясь по всему поперечному сечению проводника, как ведёт себя постоянный ток, а выводится на его поверхность. Таким образом, получается, что при включении проводника в цепь переменного тока его полезное сечение оказывается значительно меньшим, чем при включении в цепь тока постоянного. Именно поэтому его сопротивление возрастает. В физике и электротехнике это явление называется поверхностным эффектом.

То, что переменный ток распределяется по сечению проводника неравномерно, объясняется действием электродвижущей силы самоиндукции. Она индуцируется в проводнике тем магнитным полем, которое создается током, проходящим по нему. Необходимо заметить, что действие этого магнитного поля распространяется не только на окружающее проводник пространство, но и на внутреннюю его часть. По этой простой причине те слои проводника, которые располагаются ближе к его центру, находятся под воздействием большего магнитного потока, чем те слои, что располагаются ближе к его поверхности. Соответственно, электродвижущая сила самоиндукции, которая возникает во внутренних слоях, существенно больше, чем та, что образуется в слоях внешних.

Электродвижущая сила самоиндукции является существенным препятствием для изменения тока, и поэтому он будет следовать преимущественно по поверхностным слоям проводника. Необходимо также отметить, что сопротивление активных проводников в цепях переменного тока существенно зависит от частоты: чем она больше, тем выше ЭДС самоиндукции, и поэтому ток в большей степени подвергается вытеснению на поверхность.

Активное сопротивление в цепи переменного тока

Электрические лампы накаливания, печи сопротивления, бытовые нагревательные приборы, реостаты и другие приемники, где электрическая энергия преобразуется в тепловую, на схемах замещения обычно представлены только сопротивлением R.
Для схемы, изображенной на рис. 13.1, а, заданы сопротивление R и напряжение, изменяющееся по закону

u = Umsinωt

Найдём ток и мощность в цепи.

 

Ток в цепи переменного тока с активным сопротивлением.

По закону Ома найдем выражение для мгновенного тока:

где Im = Um/R — амплитуда тока

Из уравнений напряжения и тока видно, что начальные фазы обеих кривых одинаковы, т. е. напряжение и ток в цепи с сопротивлением R совпадают по фазе. Это показано на графиках и векторной диаграмме (рис. 13.1, б, б).

Действующий ток найдем, разделив амплитуду на √ 2:

Формулы (13.1) выражают закон Ома для цепи переменного тока с сопротивлением R. Внешне они ничем не отличаются от формулы для цепи постоянного тока, если переменные напряжение и ток выражены действующими величинами.

 Мгновенная мощность в цепи переменного тока с активным сопротивлением.

При переменных величинах напряжения и тока скорость преобразования электрической энергии в приемнике, т. е. его мощность, тоже изменяется. Мгновенная мощность равна произведению мгновенных величин напряжения и тока: p  = Umsinωt * Imsinωt = UmImsin2ωt

Из тригонометрии найдём 

Более наглядное представление о характере изменения мощности в цепи дает график в прямоугольной системе координат, который строится после умножения ординат кривых напряжения и тока, соответствующих ряду значений их общего аргумента — времени t. Зависимость мощности от времени — периодическая кривая (рис. 13.2). Если ось времени t поднять по чертежу на величину р = Pm√2 = UmIm√2, то относительно новой оси t’ график мощности является синусоидой с двойной частотой и начальной фазой 90°:

Таким образом, в первоначальной системе координат мгновенная, мощность равна сумме постоянной величины Р = UmIm√2 и перемен- ной р’:

р = Р + р’

Анализируя график мгновенной мощности, нетрудно заметить, что мощность в течение периода остается положительной, хотя ток и напряжение меняют свой знак. Это получается благодаря совпадению по фазе напряжения и тока.

Постоянство знака мощности говорит о том, что направление потока электрической энергии остается в течение периода неизменным, в данном случае от сети (от источника энергии) в приемник с сопротивлением R, где электрическая энергия необратимо преобразуется в другой вид энергии. В этом случае электрическая энергия называется активной.

Если R — сопротивление проводника, то в соответствии с законом Ленца — Джоуля электрическая энергия в нем преобразуется в тепло.

Активная мощность для цепи переменного тока с активным сопротивлением

Скорость преобразования электрической энергии в другой вид энергии за конечный промежуток времени, значительно больший периода изменения тока, характеризуется средней мощностью. Она равна средней мощности за период, которую называют активной.

Активная мощность — среднее арифметическое мгновенной мощности за период.

Для рассматриваемой цепи активную мощность Р нетрудно определить из графика рис. 13.2. Средняя величина мощности равна высоте прямоугольника с основанием Т, равновеликого площади, ограниченной кривой р(t) и осью абсцисс (на рисунке заштриховано).

Равенство площадей РТ = Sp выполняется, если высоту прямоугольника взять равной половине наибольшей мгновенной мощности Pm.

В этом случае часть площади Sp , находящаяся выше прямоугольника, точно укладывается в оставшуюся незаштрихованной его часть:

P = UI

Активная мощность для данной цепи равна произведению действующих величин тока и напряжения:

P = UI = I2R

С математической точки зрения активная мощность является постоянной составляющей в уравнении мгновенной мощности p(t) [см. выражение (13.2)].

Среднюю мощность за период можно найти интегрированием уравнения (13.2) в пределах периода:

Сопротивление R, определяемое из формулы (13.3) отношением активной мощности цепи к квадрату действующего тока, называется активным электрическим сопротивлением.

Переменный электрический ток. Активное, индуктивное, ёмкостное сопротивления.

Переменный электрический ток

Электромагнитные колебания, как и механические, бывают двух типов: свободные и вынужденные.

Свободные электромагнитные колебания, всегда колебания затухающие. Поэтому на практике они почти не используются. В то время, как вынужденные колебания используются везде и повсеместно. Ежедневно мы с вами можем наблюдать эти колебания.

Все наши квартиры освещены с помощью переменного тока. Переменный ток есть не что иное, как вынужденные электромагнитные колебания. Сила тока и напряжение будут меняться с течением времени согласно гармоническому закону. Колебания, например, напряжения можно обнаружить, если подать напряжение из розетки, на осциллограф.

На экране осциллографа появится синусоида. Можно вычислить частоту переменного тока. Она будет равняться частоте электромагнитных колебаний. Стандартная частота для промышленного переменного тока принята равной 50 Гц. То есть за 1 секунду направление тока в розетке меняется 50 раз.

Изменение напряжения на концах цепи будет вызывать за собой изменение силы тока в цепи колебательного контура. Следует всё же понимать, что изменение электрического поля во всей цепи не происходит мгновенно. Но так как это время, значительно меньше, чем период колебания напряжения на концах цепи, то обычно считают, что электрическое поле в цепи сразу же меняется, как меняется напряжение на концах цепи.

Переменное напряжение создается генераторами на электростанциях. Простейшим генератором можно рассматривать проволочную рамку, которая вращается в однородном магнитном поле. 

Магнитный поток, пронизывающий контур, будет постоянно меняться и будет пропорционален косинусу угла между вектором магнитной индукции и нормалью к рамке. Если рамка вращается равномерно, то угол будет пропорционален времени.

Следовательно, магнитный поток будет изменяться по гармоническому закону:

Ф = BScos(ωt)

Скорость изменения магнитного потока, взятая с обратным знаком, согласно закону ЭМИ, будет равняться ЭДС индукции.

Ei = -Ф’ = Emsin(ωt).

Если к рамке подключить колебательный контур, то угловая скорость вращения рамки определит частот колебаний напряжения на различных участках цепи и силы тока. В дальнейшем мы будем рассматривать только вынужденные электромагнитные колебания.

Они описываются следующими формулами:

u = Umsin(ωt),

u = Umcos(ωt)

Здесь Um – амплитуда колебаний напряжения. Напряжение и сила тока меняются с одинаковой частой ω. Но колебания напряжения не всегда будут совпадать с колебаниями силы тока, поэтому лучше использовать более общую формулу:

I = Imsin(ωt +φ), где Im — амплитуда колебаний силы тока, а φ – сдвиг фаз между колебаниями силы тока и напряжения.

 Активное сопротивление

Рассмотрим следующую цепь.

Она состоит из источника переменного напряжения, соединительных проводов и некоторой нагрузки. Причем индуктивность нагрузки очень мала, а сопротивление R очень велико. Эту нагрузку мы раньше называли сопротивлением. Теперь будем называть её активным сопротивлением.

Сопротивление R называют активным, так как если в цепи будет нагрузка с таким сопротивлением, цепь будет поглощать энергию, поступающую от генератора. Будем считать, что напряжение на зажимах цепи подчиняется гармоническому закону:

U = Umcos(ωt).

Мгновенное значение силы тока можем вычислить по закону Ома, оно будет пропорционально мгновенному значению напряжения.

I = u/R = Umcos(ωt)/R = Imcos(ωt).

Сделаем вывод: в проводнике с активным сопротивлением разность фаз между колебаниями напряжения и силы тока отсутствует.

Действующее значение силы тока

Амплитуда силы тока определяется по следующей формуле:

Im = Um/R.

Среднее значение квадрата силы тока за период вычисляется по следующей формуле:

i2 = (Im)2/2.

Здесь Im есть амплитуда колебания силы тока. Если мы теперь вычислим квадратный корень из среднего значения квадрата силы тока, то получим величину, которая называется действующим значением силы переменного тока. 

Для обозначения действующего значения силы тока используется буква I. То есть в виде формулы это будет выглядеть следующим образом:

I = √(i2) = Im/√2.

Действующее значение силы переменного тока будет равно силе такого постоянного тока, при котором за одинаковый промежуток времени в рассматриваемом проводнике будет выделяться столько же теплоты, сколько и при переменном токе. Для определения действующего значения напряжения используется следующая формула.

U = √(u2) = Um/√2.

Теперь подставим действующие значения силы тока и напряжения, в выражение Im = Um/R. Получим:

I = U/R.

Данное выражение является законом Ома для участка цепи с резистором, по которому течет переменный ток. Как и в случае механических колебаний, в переменном токе нас мало будут интересовать значения силы тока, напряжении в какой-то отдельный момент времени. Гораздо важнее будет знать общие характеристики колебаний — такие, как амплитуда, частота, период, действующие значения силы тока и напряжения. 

Кстати, стоит отметить, что вольтметры и амперметры, предназначенные для переменного тока, регистрируют именно действующие значения напряжения и силы тока.

Еще одним преимуществом действующих значений перед мгновенными является то, что их можно сразу использовать для вычисления значения средней мощности P переменного тока.

Для вычисления средней мощности используется следующая формула:

P = I2R = UI.

Отметим, что измерительные приборы (амперметры и вольтметры переменного тока) регистрируют именно действующие значения. Кроме того, номинальные значения напряжений и токов бытовых приборов также указываются как действующие значения. Так стандартное напряжение в цепи − 220 вольт есть действующее значение, а амплитудное значение этого напряжения равно


При изучении постоянного тока мы узнали, что он не может проходить в цепи, в которой есть конденсатор. Так как конденсатор — это две пластины, разделенные слоем диэлектрика. Для цепи постоянного тока конденсатор будет, как разрыв в цепи. Если конденсатор пропускает постоянный ток, значит, он неисправен.

В отличии от постоянного переменный ток может идти и через цепь, в которой присутствует конденсатор.

Рассмотрим, как будет меняться сила тока в цепи, содержащей конденсатор, с течением времени. При этом будем пренебрегать сопротивлением соединяющих проводов и обкладок конденсатора.

рисунок

Напряжение на конденсаторе будет равняться напряжению на концах цепи. Значит, мы можем приравнять эти две величины.

u = φ12 = q/C,   u = Umcos(ωt).

Имеем:

q/C = Umcos(ωt).

Выражаем заряд:

q = CUmcos(ωt).

Видим, что заряд будет изменяться по гармоническому закону. Сила тока — это скорость изменения заряда. Значит, если возьмем производную от заряда, получим выражение для силы тока.

I = q’ = UmCωcos(ωt+π/2).

Разность фаз между колебаниями силы тока и заряда, а также напряжения, получилась равной π/2. Получается, что колебания силы тока опережают по фазе колебания напряжения на π/2. Это представлено на рисунке.


Из уравнения колебаний силы тока получаем выражение для амплитуды силы тока:

Im = UmCω.

Введем следующее обозначение:

Xc = 1/(Cω).

Запишем следующее выражение закона Ома, используя Xc и действующие значения силы тока и напряжения:

I = U/Xc.

Xc — величина, называемая емкостным сопротивлением.

Индуктивность в цепи переменного тока будет влиять на силу переменного тока.

Рассмотрим цепь, в которой есть только катушка индуктивности. При этом значение сопротивления катушки и соединительных проводов пренебрежимо мало. 

рисунок

Выясним, как будут связаны напряжение на катушке с ЭДС самоиндукции в ней. При сопротивлении катушки равном нулю, напряженность электрического поля внутри проводника тоже будет равна нулю. Равенство нулю напряженности возможно.


Напряженности электрического поля создаваемого зарядами Eк будет соответствовать такая же по модулю и противоположно направленная напряженность вихревого электрического поля, которое появится вследствие изменения магнитного поля.

Следовательно, ЭДС самоиндукции ei будет равна по модулю и противоположна по знаку удельной работе кулоновского поля.

Следовательно:  ei = -u.

Сила тока будет изменяться по гармоническому закону: I = Im sin(ωt).

ЭДС самоиндукции будет равна:  Ei = -Li’ = -Lω Im cos(ωt).

Следовательно, напряжение будет равно:  U = Lω Im cos(ωt) = Lω Im sin(ωt+ π/2).

Отсюда значение действующего напряжения будет равняться Um = Lω Im. Видим, что между колебаниями тока и напряжения получилась разность фаз равная π/2. Следовательно, колебания силы тока отстают от колебания напряжения на π/2. Это наглядно представлено на следующем рисунке.

рисунок

Im = Um/(ωL). Введем обозначение XL = ωL. Эта величина называется индуктивное сопротивление.

Формула активного сопротивления в цепи переменного тока

Различные факторы играют важную роль для вычисления потерь в линиях транспортировки электрической энергии. Для постоянного тока вполне хватает стандартных данных об омическом сопротивлении. А вот для цепей переменной разновидности необходимо учитывать активное и индуктивное сопротивление в сочетании с емкостной проводимостью токопроводников.

Можно воспользоваться для вычислений специальными таблицами. В них представлены с большой точностью различные варианты для выполнения расчетов в сетях переменного тока. Но, чтобы быстро разобраться в специфике представленных характеристик, желательно знать природу подобного явления и его основные характеристики.

Особенности активного сопротивления

Важно! Это явление наблюдается исключительно в ситуациях с переменным током. Только он способен образовывать в кабелях оба вида противодействия.

Величина активного сопротивления обусловлена эффектом поверхностного типа. Наблюдается процесс своеобразного перемещения тока от центра к поверхности проводника. Сечение кабеля используется не полностью, а возникающее противодействие будет значительно превышать аналогичный омический показатель.

Обратим внимание на такой момент:

  1. Поверхностный эффект имеет незначительную величину в линиях из металлов, относящихся к категории цветных. Активное сопротивление приравнивают к омическому и считают его при условной температуре в +20°С, без учета фактических показателей окружающей среды. В справочниках имеются данные определения для использования в основном выражении R=r0l, с учетом того, что r0 – это номинальное значение искомой величины для 1 км провода, а l – его фактическая протяженность.
  2. А вот в стальных изделиях данный показатель намного выше. Обязательно потребуется брать во внимание, зависящее от сечения явление перемагничивания и влияние таких компонентов, как вихревые токи. На практике обычно при больших нагрузках пользуются справочными данными. При этом, само явление ослабевает в проводниках многопроволочного типа.

Индуктивное сопротивление

Созданное в ходе передачи энергии переменное магнитное поле становится источником реактивного сопротивления подобного вида. Индуктивный вариант в основном зависит от характеристик проходящего тока, диаметра и расстояния между проводами.

Само сопротивление обычно классифицируют следующим образом:

  • зависящее от параметров тока и материала — внутреннее;
  • обусловленное геометрическими особенностями линии — внешнее. В этом случае данный показатель будет постоянной величиной, не зависящей от каких-либо других факторов.

Заводы по производству кабельной продукции всегда указывают в своих каталогах информацию об индуктивном сопротивлении.

Данный параметр обычно определяется следующим выражением:

в котором индуктивный показатель для 1 км провода – , а L – протяженность.

Х километрового участка рассчитывается по следующей формуле:

Где: Dср – расстояние среднее по центральной оси имеющихся проводов, мм; d – диаметр рабочего токопроводника, мм; μт –относительная магнитная проницаемость.

Принцип действия индуктивного сопротивления линий

Реактивная и активная составляющие обуславливают полное сопротивление, которое можно представить в виде суммы квадратов каждого показателя.

Емкостная проводимость

Одним из эксплуатационных показателей остается данный параметр, обозначающий емкость между проводниками и землей, а также аналогичный показатель между самими токопроводниками.

Можно увидеть прямую зависимость рабочей емкости от уменьшения расстояния между кабелями и их сечения. Следовательно, для линий низкого напряжения данная величина всегда будет больше, чем для высокого.

Попытка применить именно подобный способ для самостоятельного выполнения будет весьма непростой задачей, ведь в нем применяются и различные конструктивные нюансы типа геометрических характеристик, и диэлектрическая проницаемость изоляционного слоя, и многие другие вводные. Следовательно, оптимальным решением будет информация из таблиц, составленных производителями для конкретной марки кабеля. В каталогах все данные приведены с учетом номинального напряжения для каждой модификации.

Данный показатель будет объективным только при полностью обесточенных приемниках электричества.

Большое значение обозначенная емкость в любой рассматриваемой конструкции имеет для точного выполнения предварительных расчетов для устройств компонентов защиты и элементов заземления.

Для кабельных магистралей:

Источник: https://uelektrika.ru/osnovy-yelektrotekhniki/aktivnoe-i-induktivnoe-soprotivleni/

Активное сопротивление: формула, от чего зависит, в чем измеряется реактивное сопротивление

Сопротивлением в электротехнике называют такую величину, которая характеризует противодействие отдельность части электрической сети или ее элементов электрическому току.

Это основано на том, что сопротивление изменяет электрическую энергию и конвертирует ее в другие типы.

Например, в сетях с переменных электротоком происходят необратимые изменения энергии и ее передача между участниками этой электроцепи.

Сопротивление как физическую величину трудно переоценить, так как она является одной из ключевых характеристик электричества в сети и прямо или пропорционально определяет силу тока и напряжение. Этот материал познакомит с такими понятиями как: активное сопротивление и реактивное сопротивление в цепи переменного тока, как проявляется зависимость активного сопротивления от частоты.

Векторное изображение полного импеданса

Какое сопротивление называется реактивным, какое активным

Активное электросопротивление — это важный параметр электрической сети, который обуславливает превращение электрической энергии, поступающей в участок электроцепи или в отдельный элетроэлемент в любой другой тип энергии: химическую, механическую, тепловую, электромагнитную. Процесс превращения при этом считаю необратимым.

Типы рассматриваемой величины и формулы ее расчета

Реактивное сопротивление по-другому называется реактансом и представляет собой сопротивляемость элементов электроцепи, которые вызывается измерением силы электротока или напряжения из-за имеющейся емкости или индуктивности этого элемента. При реактансе происходит обменный процесс между отдельным компонентом сети и источником энергии. Часто это понятие относят к простому электрическому сопротивлению, однако оно отличается некоторыми моментами.

Течение переменного электротока не зависит от типа сопротивляемости элементов и всей сети

Какие отличия

Отличия этих типов электросопротивления в том, что «внутри» активностного типа энергия не накапливается, так как она попадает в активностый элемент и отдается окружающей среде в виде другого ее типа. Это может быть тепло или механическое поднятие груза, свечение, химическая реакция, задание чему-либо скорости.

Индуктивная величина и ее формулы

Важно! Преданная электроэлементу с активностным электросопротивлением энергия преображается и конвертируется, но не возвращается в сеть.

Сопротивляемость же реактивная, наоборот, копит энергию внутри себя за ¼ всего периода синусоидального электротока, а за следующую четверть возвращает ее обратно в сеть. То есть, в окружающую среду полученная энергия не передается.

Комплексная сопротивляемость отдельного элетроэлемента сети R

В активностном типе фазы электрических токов и напряжения совпадают, следовательно, выделяется некоторое количество электроэнергии. В реактивном виде фазы электротока и напряжения расходятся, поэтому энергия передается обратно. Это во многом объясняет то, что активностные электроэлементы нагреваются, а реактивные — нет.

Активная сопротивляемость в цепи переменного синусоидального тока

От чего зависит активное сопротивление

Активное электросопротивление зависит от сечения проводника. Это значит, что полезным сечением при электротоке с высокой частотой будет только тонкий наружный слой проводника. Из этого исходит также то, что активностное электросопротивление только возрастает с увеличением частоты электротока переменного типа.

Для того чтобы уменьшить поверхностный эффект проводника, по которому течет электроток высокой частоты, его изготавливают трубчатым и покрывают напылением металла, хорошо проводящего электрический ток, например, серебром.

Схема косвенного метода амперметра, вольтметра и ваттметра

В чем измеряется реактивное сопротивление

Само по себе, явление реактанса характерно только для цепей с электрическим током переменного типа. Обозначается оно латинской буквой «X» и измеряется в Омах.

В отличие от активностного варианта, реактанс может иметь как положительное, так и отрицательное значение. Знак «+» или «-» соответствует знаку, по которому сдвигается фаза электротока и напряжения.

Знак положительный, когда ток отстает от напряжения и отрицателен, когда кот опережает напряжение.

Важно! Абсолютно чистое реактивное электросопротивление имеет сдвиг фазы на ± 180/2. То есть, фаза «двигается» на π/2.

Примером активной сопротивляемости — линия электропередач

Как правильно измерять сопротивление

При работе с радиоаппаратурой иногда требуется измерять не только активностное, но и реактивное электросопротивление (индуктивность и емкость). Для измерений применяют косвенный метод использования мультиметра, а более точные значения получают при мостовом методе.

Активом сопротивляемости может выступать любой резистор

Косвенный метод наиболее прост в своей реализации, так как не требует дополнительных схем включения. Одна требуется наличие трех отдельных приборов: амперметра, вольтметра и ваттметра.

Если измерить напряжение и силу электротока в цепи, то можно получить полное электросопротивление: Z=U*I  После измерения активностной мощности P, можно получить величину активного сопротивления отдельного элемента: R= P/I².

Обмотка трансформатора — один из примеров актива по превращению электроэнергии

Области проявления

Реактанс электросопротивления проявляется в емкости и индукции. Первое обуславливается наличием емкости проводниках и обмотках или включением в электрическую цепь переменного тока различных конденсаторов. Чем выше емкость потребителя и угловой частоты сигнала электротока, тем меньше емкостная характеристика.

Сопротивляемость, которую оказывает проводник переменному току и электродвижущей силе самоиндукции, называется индуктивным. Оно зависит от индуктивности потребителя.

Чем выше его индуктивность и выше частота переменного электротока, тем выше индуктивное электросопротивление.

Выражается оно формулой: xl = ωL, где xl — это электросопротивление индукции, L — индуктивность, а ω — угловая частота тока.

Емкостный реактанс электросопротивление проявляется, например, в конденсаторе, который накапливает электроэнергию в виде электромагнитного поля между своими обкладками. Индуктивное электросопротивление можно наблюдать в дросселе, который накапливает энергию в виде магнитного поля внутри своей обмотки.

Активностным же электросопротивлением может обладать любой резистор, линии электропередач, обмотки трансформатора или электрического двигателя.

Индукция ЭДС может наблюдаться в дросселе

Таким образом, активный резист и реактанс во многом отличаются друг от друга не только разницей по названию, но и по физическим свойствам. Первый вид превращает электроэнергию в другой вид и отдает ее в окружающую среду. Второй же — возвращает ее обратно в электросеть.

Источник: https://rusenergetics.ru/polezno-znat/aktivnoe-soprotivlenie

Активное сопротивление. Действующие значения силы тока и напряжения — Класс!ная физика

«Физика — 11 класс»

Активное сопротивление

Сила тока в цепи с резистором

Есть цепь, состоящая из соединительных проводов и нагрузки с малой индуктивностью и большим сопротивлением R.

Сопротивление R называется активным сопротивлением, т.к. при наличии нагрузки, обладающей этим сопротивлением, цепь поглощает энергию, поступающую от генератора. Эта энергия превращается во внутреннюю энергию проводников — они нагреваются.

  • Напряжение на зажимах цепи меняется по гармоническому закону:
  • u = Um cos ωt
  • Мгновенное значение силы тока прямо пропорционально мгновенному значению напряжения. По закону Ома мгновенное значение силы тока:

В проводнике с активным сопротивлением колебания силы тока совпадают по фазе с колебаниями напряжения, а амплитуда силы тока определяется равенством

Мощность в цепи с резистором

В цепи переменного тока промышленной частоты (v = 50 Гц) сила тока и напряжение меняются. При прохождении тока по проводнику, например по нити электрической лампочки, количество выделенной энергии также будет меняться во времени.

  1. Мощность в цепи постоянного тока на участке с сопротивлением R определяется формулой
  2. Р = I2R
  3. Мгновенная мощность в цепи переменного тока на участке, имеющем активное сопротивление R, определяется формулой
  4. Р = i2R

График зависимости мгновенной мощности от времени (рис.а):

Согласно графику (рис.б) среднее за период значение cos 2ωt равно нулю, а значит равно нулю второе слагаемое в формуле для среднего значения мощности за период.

Действующие значения силы тока и напряжения

Среднее за период значение квадрата силы тока:

Величина, равная квадратному корню из среднего значения квадрата силы тока, называется действующим значением силы переменного тока. Действующее значение силы переменного тока обозначается через I:

Действующее значение силы переменного тока равно силе такого постоянного тока, при котором в проводнике выделяется то же количество теплоты, что и при переменном токе за то же время.

Действующее значение переменного напряжения определяется аналогично:

Закон Ома для участка цепи переменного тока с резистором в действующих значениях:

В случае электрических колебаний важны общие характеристики колебаний, такие, как амплитуда, период, частота, действующие значения силы тока и напряжения, средняя мощность. Именно действующие значения силы тока и напряжения регистрируют амперметры и вольтметры переменного тока.

  • Действующие значения непосредственно определяют среднее значение мощности Р переменного тока:
  • р = I2R = UI.
  • Итак: Колебания силы тока в цепи с резистором совпадают по фазе с колебаниями напряжения, а мощность определяется действующими значениями силы тока и напряжения.
  • Источник: «Физика — 11 класс», учебник Мякишев, Буховцев, Чаругин

Следующая страница «Конденсатор в цепи переменного тока» Назад в раздел «Физика — 11 класс, учебник Мякишев, Буховцев, Чаругин»

Электромагнитные колебания. Физика, учебник для 11 класса — Класс!ная физика

Свободные и вынужденные электромагнитные колебания. Колебательный контур. Превращение энергии при электромагнитных колебаниях — Аналогия между механическими и электромагнитными колебаниями — Уравнение, описывающее процессы в колебательном контуре.

Период свободных электрических колебаний — Переменный электрический ток — Активное сопротивление.

Действующие значения силы тока и напряжения — Конденсатор в цепи переменного тока — Катушка индуктивности в цепи переменного тока — Резонанс в электрической цепи — Генератор на транзисторе. Автоколебания — Краткие итоги главы

Источник: http://class-fizika.ru/11_27.html

Цепь переменного тока с активным сопротивлением

Когда в электрическую цепь переменного тока подключается активное сопротивление R, то под воздействием разницы потенциалов источника в цепи начинает течь ток I. В тех случаях, когда изменение напряжения происходит по синусоидальному закону, который выражается, как u = Um sin ωt, то изменение тока i также идет по синусоиде:

Активное сопротивление

i = Im sin ωt

При этом

Так что получается, что изменение напряжения и тока происходят по одинаковым законам. При этом через нулевое значение они проходят одновременно и своих максимальных значений также достигают одновременно. Из этого следует, что когда в электрическую цепь переменного тока подключается активное сопротивление R, то напряжение и ток совпадают по фазе.

Мощность, ток, напряжение

Если взять равенство Im = Um / R и каждую из его частей разделить на √2, то в итоге получится ни что иное, как закон Ома, применимый для той цепи, которая рассматривается: I = U/R.

Таким образом, получается, что это основополагающий закон для той цепи, которая имеет в своем составе только активное сопротивление, с точки зрения математики имеет такую же форму, что и для цепи тока постоянного.

Такой показатель, как электрическая мощность P для цепи, имеющей в своем составе активное сопротивление, равняется произведению мгновенного значения напряжения U на мгновенное значение силы тока i в любой момент времени.

Из этого следует, что в цепях переменного тока, в отличие от цепей тока постоянного, мгновенная мощность P – величина непостоянная, а ее изменение происходит по кривой. Для того чтобы получить ее графическое представление, необходимо ординаты кривых напряжения U и силы тока i перемножить при разных углах ωt.

Мощность изменяется по отношению к изменению тока с двойной частотой ωt. Это означает, что половине периода изменения напряжения и тока соответствует один период изменения мощности. Следует заметить, что абсолютно все значения, которые может принимать мощность, являются положительными величинами.

С точки зрения физики это означает, что от источника к приемнику передается энергия. Своих максимальных значений мощность достигает тогда, когда ωt = 270° и ωt = 90°.

В практическом отношении о той энергии W, которую создает электрический ток, принято судить по средней мощности, выражаемой формулой Рср = Р, а не по мощности максимальной. Ее можно определить, перемножив на время протекания тока среднее значение мощности W = Pt.

  • Относительно линии АБ, соответствующей среднему значению мощности P, кривая мгновенной мощности симметрична. По этой причине
  • P = Pmax / 2 = UI
  • Если использовать закон Ома, то можно выразить активную мощность в следующем виде:
  • P = I2R или P = U2/R.
  • Специалисты в области электротехники ту среднюю мощность, которую потребляет активное сопротивление, чаще всего именуют или просто мощностью, или активной мощностью, а для ее обозначения используется буква P.

Необходимо особо отметить такую особенность проводников, включенных в сеть переменного тока: их активное сопротивление во всех случаях оказывается больше, чем если бы они были включены в сеть тока постоянного.

Причина этого состоит в том, что переменный ток не протекает равномерно распределяясь по всему поперечному сечению проводника, как ведёт себя постоянный ток, а выводится на его поверхность.

Таким образом, получается, что при включении проводника в цепь переменного тока его полезное сечение оказывается значительно меньшим, чем при включении в цепь тока постоянного. Именно поэтому его сопротивление возрастает. В физике и электротехнике это явление называется поверхностным эффектом.

То, что переменный ток распределяется по сечению проводника неравномерно, объясняется действием электродвижущей силы самоиндукции. Она индуцируется в проводнике тем магнитным полем, которое создается током, проходящим по нему.

Необходимо заметить, что действие этого магнитного поля распространяется не только на окружающее проводник пространство, но и на внутреннюю его часть. По этой простой причине те слои проводника, которые располагаются ближе к его центру, находятся под воздействием большего магнитного потока, чем те слои, что располагаются ближе к его поверхности.

Соответственно, электродвижущая сила самоиндукции, которая возникает во внутренних слоях, существенно больше, чем та, что образуется в слоях внешних.

Электродвижущая сила самоиндукции является существенным препятствием для изменения тока, и поэтому он будет следовать преимущественно по поверхностным слоям проводника.

Необходимо также отметить, что сопротивление активных проводников в цепях переменного тока существенно зависит от частоты: чем она больше, тем выше ЭДС самоиндукции, и поэтому ток в большей степени подвергается вытеснению на поверхность.

Источник: http://selectelement.ru/basic-concepts/ac-active-resistance.php

Активное и реактивное сопротивление

В электротехнике понятие сопротивления представляет собой величину, за счет которой определенная часть цепи может противодействовать электрическому току. Она образуется за счет изменения и перехода электроэнергии в другое энергетическое состояние.

Данное явление присуще только переменному току, когда в сети образуется активное и реактивное сопротивление, выражающееся в необратимом изменении энергии или передаче этой энергии между отдельными компонентами электрической цепи.

В случае необратимых изменений электроэнергии сопротивление будет считаться активным, а при наличии обменных процессов – реактивным.

Основные различия между активным и реактивным сопротивлением

Когда электрический ток проходит через элементы с активным сопротивлением, происходят необратимые потери выделяемой мощности. Типичным примером служит электрическая плита, где в процессе работы происходят необратимые превращения электричества в тепловую энергию. То же самое происходит с резистором, в котором тепло выделяется, но обратно в электроэнергию не превращается.

Реактивное сопротивление возникает в тех случаях, когда переменный ток проходит через так называемые реактивные элементы, обладающие индуктивностью и емкостью.

Первое свойство характерно для катушки индуктивности без учета активного сопротивления ее обмотки. В данном случае причиной появления реактивного сопротивления считается ЭДС самоиндукции.

В зависимости от частоты тока, при ее возрастании, наблюдается и одновременный рост сопротивления, что отражается в формуле xl = wL.

Закон Ома для участка цепи

Реактивное сопротивление конденсатора зависит от емкости. Оно будет уменьшаться при увеличении частоты тока, поэтому данное свойство широко используется в электронике для выполнения регулировочных функций. В этом случае для расчетов используется формула xc = 1/wC.

В графике это выражение выглядит в виде треугольника сопротивлений, где реактивное и активное сопротивление соответствуют катетам, а полное сопротивление или импеданс – гипотенузе.

Индуктивное сопротивление

Реактивное сопротивление подразделяется на два основных вида – индуктивное и емкостное.

При рассмотрении первого варианта следует отметить возникновение в индуктивной обмотке магнитного поля под действием переменного тока.

В результате, в ней образуется ЭДС самоиндукции, направленной против движения тока при его росте, и по ходу движения при его уменьшении.

Таким образом, при всех изменениях тока и наличии взаимосвязей, ЭДС оказывает на него противоположное действие и приводит к созданию индуктивного сопротивления катушки.

Под влиянием ЭДС самоиндукции энергия магнитного поля обмотки возвращается в электрическую цепь. То есть, между источником питания и обмоткой происходит своеобразный обмен энергией. Это дает основание полагать, что катушка индуктивности обладает реактивным сопротивлением.

В качестве типичного примера можно рассмотреть действие реактивного сопротивления в трансформаторе. Данное устройство имеет общий магнитопровод, с расположенными на нем двумя обмотками или более, имеющими общую зависимость. На одну из них поступает электроэнергия из внешнего источника, а из другой выходит уже трансформированный ток.

Под действием первичного тока, проходящего по катушке, в магнитопроводе и вокруг него происходит наведение магнитного потока. В результате пересечения витков вторичной обмотки, в ней формируется вторичный ток.

При невозможности создания идеальной конструкции трансформатора, магнитный поток будет частично уходить в окружающую среду, что приведет к возникновению потерь.

От них зависит величина реактивного сопротивления рассеяния, которая совместно с активной составляющей образуют комплексное сопротивление, называемое электрическим импедансом трансформатора.

Конденсатор в цепи переменного тока

Емкостное сопротивление

В цепи, содержащей емкость и источник переменного тока происходят изменения заряда. Такой емкостью обладают конденсаторы, обладающие максимальной энергией при полном заряде. Напряжение емкости создает сопротивление, противодействующее течению переменного тока, которое считается реактивным. В результате взаимодействия, конденсатор и источник тока постоянно обмениваются энергией.

В конструкцию конденсатора входят токопроводящие пластины в количестве двух и более штук, разделенных слоями диэлектрика. Такое разделение не позволяет постоянному току проходить через конденсатор. Переменный ток может проходить через емкостное устройство, отклоняясь при этом от своей первоначальной величины.

Изменения переменного тока происходят под влиянием емкостного сопротивления. Чтобы лучше понять схему работы, найдем и рассмотрим принцип действия данного явления. Переменное напряжение, приложенное к конденсатору, изменяется в форме синусоиды.

Под его воздействием на обкладках наблюдается всплеск, одновременно здесь накапливаются заряды электроэнергии с противоположными знаками. Их общее количество ограничено емкостью устройства и его габаритами.

Чем выше емкость устройства, тем больше времени требуется на зарядку.

В момент изменения полупериода колебания, напряжение на обкладках конденсатора меняет свою полярность на противоположное значение, потенциалы также изменяются, а заряды пластин перезаряжаются.

За счет этого удается создать течение первичного тока и находить способ противодействовать его прохождению, при уменьшении величины и сдвиге угла.

Зарядка обкладок позволяет току, проходящему через конденсатор, опережать напряжение на 90.

Компенсация реактивной мощности

С помощью электрических сетей осуществляется передача электроэнергии на значительные расстояния.

В большинстве случаев она используется для питания электродвигателей, имеющих высокое индуктивное сопротивление и большое количество резистивных элементов. К потребителям поступает полная мощность, которая делится на активную и реактивную.

В первом случае с помощью активной мощности совершается полезная работа, а во втором – происходит нагрев трансформаторных обмоток и электродвигателей.

Сколько миллиампер в ампере

Под действием реактивной составляющей, возникающей на индуктивных сопротивлениях, существенно понижается качество электроэнергии. Противостоять ее вредному воздействию помогает комплекс мероприятий по компенсации с использованием конденсаторных батарей. За счет емкостного сопротивления удается понизить косинус угла φ.

Компенсирующие устройства применяются на подстанциях, от которых электричество поступает к проблемным потребителям. Этот способ дает положительные результаты не только в промышленности, но и на бытовых объектах, снижая нагрузку на оборудование.

Источник: https://electric-220.ru/news/aktivnoe_i_reaktivnoe_soprotivlenie/2017-12-23-1414

Активное сопротивление в цепи переменного тока

Электрические лампы накаливания, печи сопротивления, бытовые нагревательные приборы, реостаты и другие приемники, где электрическая энергия преобразуется в тепловую, на схемах замещения обычно представлены только сопротивлением R. Для схемы, изображенной на рис. 13.1, а, заданы сопротивление R и напряжение, изменяющееся по закону

u = Umsinωt

Найдём ток и мощность в цепи.

Ток в цепи переменного тока с активным сопротивлением

По закону Ома найдем выражение для мгновенного тока:

где Im = Um/R — амплитуда тока

Из уравнений напряжения и тока видно, что начальные фазы обеих кривых одинаковы, т. е. напряжение и ток в цепи с сопротивлением R совпадают по фазе. Это показано на графиках и векторной диаграмме (рис. 13.1, б, б).

Действующий ток найдем, разделив амплитуду на √ 2:

Формулы (13.1) выражают закон Ома для цепи переменного тока с сопротивлением R. Внешне они ничем не отличаются от формулы для цепи постоянного тока, если переменные напряжение и ток выражены действующими величинами.

 Мгновенная мощность в цепи переменного тока с активным сопротивлением

При переменных величинах напряжения и тока скорость преобразования электрической энергии в приемнике, т. е. его мощность, тоже изменяется. Мгновенная мощность равна произведению мгновенных величин напряжения и тока: p  = Umsinωt * Imsinωt = UmImsin2ωt

Из тригонометрии найдём 

Более наглядное представление о характере изменения мощности в цепи дает график в прямоугольной системе координат, который строится после умножения ординат кривых напряжения и тока, соответствующих ряду значений их общего аргумента — времени t.

 Зависимость мощности от времени — периодическая кривая (рис. 13.2).

Если ось времени t поднять по чертежу на величину р = Pm√2 = UmIm√2, то относительно новой оси t’ график мощности является синусоидой с двойной частотой и начальной фазой 90°:

Таким образом, в первоначальной системе координат мгновенная, мощность равна сумме постоянной величины Р = UmIm√2 и перемен- ной р’:

р = Р + р’

Анализируя график мгновенной мощности, нетрудно заметить, что мощность в течение периода остается положительной, хотя ток и напряжение меняют свой знак. Это получается благодаря совпадению по фазе напряжения и тока.

Постоянство знака мощности говорит о том, что направление потока электрической энергии остается в течение периода неизменным, в данном случае от сети (от источника энергии) в приемник с сопротивлением R, где электрическая энергия необратимо преобразуется в другой вид энергии. В этом случае электрическая энергия называется активной.

Если R — сопротивление проводника, то в соответствии с законом Ленца — Джоуля электрическая энергия в нем преобразуется в тепло.

Активная мощность для цепи переменного тока с активным сопротивлением

Скорость преобразования электрической энергии в другой вид энергии за конечный промежуток времени, значительно больший периода изменения тока, характеризуется средней мощностью. Она равна средней мощности за период, которую называют активной.

Активная мощность — среднее арифметическое мгновенной мощности за период.

Для рассматриваемой цепи активную мощность Р нетрудно определить из графика рис. 13.2. Средняя величина мощности равна высоте прямоугольника с основанием Т, равновеликого площади, ограниченной кривой р(t) и осью абсцисс (на рисунке заштриховано).

  • Равенство площадей РТ = Sp выполняется, если высоту прямоугольника взять равной половине наибольшей мгновенной мощности Pm.
  • В этом случае часть площади Sp , находящаяся выше прямоугольника, точно укладывается в оставшуюся незаштрихованной его часть:
  • P = UI
  • Активная мощность для данной цепи равна произведению действующих величин тока и напряжения:
  • P = UI = I2R = U2R

С математической точки зрения активная мощность является постоянной составляющей в уравнении мгновенной мощности p(t) [см. выражение (13.2)].

Среднюю мощность за период можно найти интегрированием уравнения (13.2) в пределах периода:

Сопротивление R, определяемое из формулы (13.3) отношением активной мощности цепи к квадрату действующего тока, называется активным электрическим сопротивлением.

Источник: https://electrikam.com/aktivnoe-soprotivlenie-v-cepi-peremennogo-toka/

Активное и реактивное сопротивление. Треугольник сопротивлений

Активное и реактивное сопротивление — сопротивлением в электротехнике называется величина, которая характеризует противодействие части цепи электрическому току. Это сопротивление образовано путем изменения электрической энергии в другие типы энергии. В сетях переменного тока имеется необратимое изменение энергии и передача энергии между участниками электрической цепи.

При необратимом изменении электроэнергии компонента цепи в другие типы энергии, сопротивление элемента является активным. При осуществлении обменного процесса электроэнергией между компонентом цепи и источником, то сопротивление реактивное.

В электрической плите электроэнергия необратимо преобразуется в тепло, вследствие этого электроплита имеет активное сопротивление, так же как и элементы, преобразующие электричество в свет, механическое движение и т.д.

В индуктивной обмотке переменный ток образует магнитное поле. Под воздействием переменного тока в обмотке образуется ЭДС самоиндукции, которая направлена навстречу току при его увеличении, и по ходу тока при его уменьшении. Поэтому, ЭДС оказывает противоположное действие изменению тока, создавая индуктивное сопротивление катушки.

С помощью ЭДС самоиндукции осуществляется возвращение энергии магнитного поля обмотки в электрическую цепь. В итоге обмотка индуктивности и источник питания производят обмен энергией. Это можно сравнить с маятником, который при колебаниях преобразует потенциальную и кинетическую энергию. Отсюда следует, что сопротивление индуктивной катушки имеет реактивное сопротивление.

Самоиндукция не образуется в цепи постоянного тока, и индуктивное сопротивление отсутствует. В цепи емкости и источника переменного тока изменяется заряд, значит между емкостью и источником тока протекает переменный ток. При полном заряде конденсатора его энергия наибольшая.

В цепи напряжение емкости создает противодействие течению тока своим сопротивлением, и называется реактивным. Между конденсатором и источником происходит обмен энергией.

После полной зарядки емкости постоянным током напряжение его поля выравнивает напряжение источника, поэтому ток равен нулю.

Конденсатор и катушка в цепи переменного тока работают некоторое время в качестве потребителя энергии, когда накапливают заряд. И также работают в качестве генератора при возвращении энергии обратно в цепь.

Если сказать простыми словами, то активное и реактивное сопротивление – это противодействие току снижения напряжения на элементе схемы. Величина снижения напряжения на активном сопротивлении имеет всегда встречное направление, а на реактивной составляющей – попутно току или навстречу, создавая сопротивление изменению тока

Настоящие элементы цепи на практике имеют все три вида сопротивления сразу. Но иногда можно пренебречь некоторыми из них ввиду незначительных величин. Например, емкость имеет только емкостное сопротивление (при пренебрежении потерь энергии), лампы освещения имеют только активное (омическое) сопротивление, а обмотки трансформатора и электромотора – индуктивное и активное.

Активное сопротивление

В цепи действия напряжения и тока, создает противодействие, снижения напряжения на активном сопротивлении. Падение напряжения, созданное током и оказывающее противодействие ему, равно активному сопротивлению.

При протекании тока по компонентам с активным сопротивлением, снижение мощности становится необратимым. Можно рассмотреть резистор, на котором выделяется тепло. Выделенное тепло не превращается обратно в электроэнергию. Активное сопротивление, также может иметь линия передачи электроэнергии, соединительные кабели, проводники, катушки трансформаторов, обмотки электромотора и т.д.

  • Отличительным признаком элементов цепи, которые обладают только активной составляющей сопротивления, является совпадение напряжения и тока по фазе. Это сопротивление вычисляется по формуле:
  • R = U/I, где R – сопротивление элемента, U – напряжение на нем, I – сила тока, протекающего через элемент цепи.
  • На активное сопротивление влияют свойства и параметры проводника: температура, поперечное сечение, материал, длина.

Реактивное сопротивление

Тип сопротивления, определяющий соотношение напряжения и тока на емкостной и индуктивной нагрузке, не обусловленное количеством израсходованной электроэнергии, называется реактивным сопротивлением. Оно имеет место только при переменном токе, и может иметь отрицательное и положительное значение, в зависимости от направления сдвига фаз тока и напряжения. При отставании тока от напряжения величина реактивной составляющей сопротивления имеет положительное значение, а если отстает напряжение от тока, то реактивное сопротивление имеет знак минус.

Активное и реактивное сопротивление, свойства и разновидности

Рассмотрим два вида этого сопротивления: емкостное и индуктивное. Для трансформаторов, соленоидов, обмоток генераторов и моторов характерно индуктивное сопротивление. Емкостный вид сопротивления имеют конденсаторы. Чтобы определить соотношение напряжения и тока, нужно знать значение обоих видов сопротивления, которое оказывает проводник.

Реактивное сопротивление образуется при помощи снижения реактивной мощности, затраченной на образование магнитного поля в цепи. Снижение реактивной мощности создается путем подключения к трансформатору прибора с активным сопротивлением.

Конденсатор, подключенный в цепь, успевает накопить только ограниченную часть заряда перед изменением полярности напряжения на противоположный. Поэтому ток не снижается до нуля, так как при постоянном токе. Чем ниже частота тока, тем меньше заряда накопит конденсатор, и будет меньше создавать противодействие току, что образует реактивное сопротивление.

Иногда цепь имеет реактивные компоненты, но в результате реактивная составляющая равна нулю. Это подразумевает равенство фазного напряжения и тока. В случае отличия от нуля реактивного сопротивления, между током и напряжением образуется разность фаз.

Катушка имеет индуктивное сопротивлением в схеме цепи переменного тока. В идеальном виде ее активное сопротивление не учитывают. Индуктивное сопротивление образуется с помощью ЭДС самоиндукции. При повышении частоты тока возрастает и индуктивное сопротивление.

На индуктивное сопротивление катушки оказывает влияние индуктивность обмотки и частота в сети.

Конденсатор образует реактивное сопротивление из-за наличия емкости. При возрастании частоты в сети его емкостное противодействие (сопротивление) снижается. Это дает возможность активно его применять в электронной промышленности в виде шунта с изменяемой величиной.

Треугольник сопротивлений

Схема цепи, подключенной к переменному току, имеет полное сопротивление, которое можно определить в виде суммы квадратов реактивного и активного сопротивлений.

Если изобразить это выражение в виде графика, то получится треугольник сопротивлений. Он образуется, если рассчитать последовательную цепь всех трех видов сопротивлений.

По этому треугольному графику можно увидеть, что катеты представляют собой активное и реактивное сопротивление, а гипотенуза является полным сопротивлением.

Похожие темы:

Источник: https://electrosam.ru/glavnaja/jelektrotehnika/aktivnoe-i-reaktivnoe-soprotivlenie/

Что такое активное сопротивление переменного тока?

Активное и реактивное сопротивление — сопротивлением в электротехнике называется величина, которая характеризует противодействие части цепи электрическому току. Это сопротивление образовано путем изменения электрической энергии в другие типы энергии. В сетях переменного тока имеется необратимое изменение энергии и передача энергии между участниками электрической цепи.

При необратимом изменении электроэнергии компонента цепи в другие типы энергии, сопротивление элемента является активным. При осуществлении обменного процесса электроэнергией между компонентом цепи и источником, то сопротивление реактивное.

В электрической плите электроэнергия необратимо преобразуется в тепло, вследствие этого электроплита имеет активное сопротивление, так же как и элементы, преобразующие электричество в свет, механическое движение и т.д.

В индуктивной обмотке переменный ток образует магнитное поле. Под воздействием переменного тока в обмотке образуется ЭДС самоиндукции, которая направлена навстречу току при его увеличении, и по ходу тока при его уменьшении. Поэтому, ЭДС оказывает противоположное действие изменению тока, создавая индуктивное сопротивление катушки.

С помощью ЭДС самоиндукции осуществляется возвращение энергии магнитного поля обмотки в электрическую цепь.

В итоге обмотка индуктивности и источник питания производят обмен энергией. Это можно сравнить с маятником, который при колебаниях преобразует потенциальную и кинетическую энергию. Отсюда следует, что сопротивление индуктивной катушки имеет реактивное сопротивление.

Самоиндукция не образуется в цепи постоянного тока, и индуктивное сопротивление отсутствует. В цепи емкости и источника переменного тока изменяется заряд, значит между емкостью и источником тока протекает переменный ток. При полном заряде конденсатора его энергия наибольшая.

В цепи напряжение емкости создает противодействие течению тока своим сопротивлением, и называется реактивным. Между конденсатором и источником происходит обмен энергией.

После полной зарядки емкости постоянным током напряжение его поля выравнивает напряжение источника, поэтому ток равен нулю.

Конденсаторикатушкав цепи переменного тока работают некоторое время в качестве потребителя энергии, когда накапливают заряд. И также работают в качестве генератора при возвращении энергии обратно в цепь.

Если сказать простыми словами, то активное и реактивное сопротивление – это противодействие току снижения напряжения на элементе схемы. Величина снижения напряжения на активном сопротивлении имеет всегда встречное направление, а на реактивной составляющей – попутно току или навстречу, создавая сопротивление изменению тока.

Настоящие элементы цепи на практике имеют все три вида сопротивления сразу. Но иногда можно пренебречь некоторыми из них ввиду незначительных величин. Например, емкость имеет только емкостное сопротивление (при пренебрежении потерь энергии), лампы освещения имеют только активное (омическое) сопротивление, а обмотки трансформатора и электромотора – индуктивное и активное.

Содержание

  • 1 Активное сопротивление
  • 2 Реактивное сопротивление
  • 3 Тип сопротивления, определяющий соотношение напряжения и тока на емкостной и индуктивной нагрузке, не обусловленное количеством израсходованной электроэнергии, называется реактивным сопротивлением. Оно имеет место только при переменном токе, и может иметь отрицательное и положительное значение, в зависимости от направления сдвига фаз тока и напряжения. При отставании тока от напряжения величина реактивной составляющей сопротивления имеет положительное значение, а если отстает напряжение от тока, то реактивное сопротивление имеет знак минус.
  • 4 Активное и реактивное сопротивление, свойства и разновидности
  • 5 Треугольник сопротивлений
  • 6 Если изобразить это выражение в виде графика, то получится треугольник сопротивлений. Он образуется, если рассчитать последовательную цепь всех трех видов сопротивлений. По этому треугольному графику можно увидеть, что катеты представляют собой активное и реактивное сопротивление, а гипотенуза является полным сопротивлением.
  • 7 Похожие темы:
  • 8 Активное сопротивление
  • 9 Реактивное сопротивление
  • 10 Конденсатор обладает реактивным сопротивлением благодаря своей ёмкости. Его сопротивление с увеличением частоты тока уменьшается, что позволяет его активно использовать в электронике в качестве шунта переменной составляющей тока. Сопротивление конденсатора можно рассчитать по формуле
  • 11 Треугольник сопротивлений

Активное сопротивление

В цепи действия напряжения и тока, создает противодействие, снижения напряжения на активном сопротивлении. Падение напряжения, созданное током и оказывающее противодействие ему, равно активному сопротивлению.

При протекании тока по компонентам с активным сопротивлением, снижение мощности становится необратимым. Можно рассмотреть резистор, на котором выделяется тепло.Выделенное тепло не превращается обратно в электроэнергию.

Активное сопротивление, также может иметь линия передачи электроэнергии, соединительные кабели, проводники, катушки трансформаторов, обмотки электромотора и т. д.Отличительным признаком элементов цепи, которые обладают только активной составляющей сопротивления, является совпадение напряжения и тока по фазе. Это сопротивление вычисляется по формуле:R = U/I, где R– сопротивление элемента,U– напряжение на нем, I– сила тока, протекающего через элемент цепи.На активное сопротивление влияют свойства и параметры проводника: температура, поперечное сечение, материал, длина.

Реактивное сопротивление

Тип сопротивления, определяющий соотношение напряжения и тока на емкостной и индуктивной нагрузке, не обусловленное количеством израсходованной электроэнергии, называется реактивным сопротивлением. Оно имеет место только при переменном токе, и может иметь отрицательное и положительное значение, в зависимости от направления сдвига фаз тока и напряжения. При отставании тока от напряжения величина реактивной составляющей сопротивления имеет положительное значение, а если отстает напряжение от тока, то реактивное сопротивление имеет знак минус.

Активное и реактивное сопротивление, свойства и разновидности

Рассмотрим два вида этого сопротивления: емкостное и индуктивное.

Для трансформаторов, соленоидов, обмоток генераторов и моторов характерно индуктивное сопротивление. Емкостный вид сопротивления имеют конденсаторы. Чтобы определить соотношение напряжения и тока, нужно знать значение обоих видов сопротивления, которое оказывает проводник.

Реактивное сопротивление образуется при помощи снижения реактивной мощности, затраченной на образование магнитного поля в цепи. Снижение реактивной мощности создается путем подключения к трансформатору прибора с активным сопротивлением.

Конденсатор, подключенный в цепь, успевает накопить только ограниченную часть заряда перед изменением полярности напряжения на противоположный. Поэтому ток не снижается до нуля, так как при постоянном токе. Чем ниже частота тока, тем меньше заряда накопит конденсатор, и будет меньше создавать противодействие току, что образует реактивное сопротивление.

Иногда цепь имеет реактивные компоненты, но в результате реактивная составляющая равна нулю. Это подразумевает равенство фазного напряжения и тока. В случае отличия от нуля реактивного сопротивления, между током и напряжением образуется разность фаз.

Катушка имеет индуктивное сопротивлением в схеме цепи переменного тока.

В идеальном виде ее активное сопротивление не учитывают. Индуктивное сопротивление образуется с помощью ЭДС самоиндукции. При повышении частоты тока возрастает и индуктивное сопротивление.

На индуктивное сопротивление катушки оказывает влияние индуктивность обмотки и частота в сети.

Конденсатор образует реактивное сопротивление из-за наличия емкости. При возрастании частоты в сети его емкостное противодействие (сопротивление) снижается. Это дает возможность активно его применять в электронной промышленности в виде шунта с изменяемой величиной.

Треугольник сопротивлений

Схема цепи, подключенной к переменному току, имеет полное сопротивление, которое можно определить в виде суммы квадратов реактивного и активного сопротивлений.

Если изобразить это выражение в виде графика, то получится треугольник сопротивлений. Он образуется, если рассчитать последовательную цепь всех трех видов сопротивлений.

По этому треугольному графику можно увидеть, что катеты представляют собой активное и реактивное сопротивление, а гипотенуза является полным сопротивлением.

Похожие темы:

[ads-pc-1]

Ток и напряжение.

При включении в цепь переменного тока активного сопротивления R (рис. 175, а) напряжение и источника создает в цепи ток i. Если напряжение и изменяется по синусоидальному закону u = Uтsin ?t, то ток i также изменяется синусоидально:

i = Iтsin ?t

При этом

Iт= Uт/ R

Таким образом, ток и напряжение изменяются по одному и тому же закону; они одновременно достигают своих максимальных значений и одновременно проходят через нуль (рис.

175,б). Следовательно,при включении в цепь переменного тока активного сопротивления ток и напряжение совпадают по фазе (рис. 175, в).

Если обе части равенства Iт= Uт/ Rразделить на ?2, то получим выражение закона Ома для рассматриваемой цепи для действующих значений напряжения и тока:

I = U / R

Следовательно, для цепи переменного тока, содержащей только активное сопротивление, этот закон имеет такую же математическую форму, как и для цепи постоянного тока.

Электрическая мощность.Электрическая мощность р в цепи с активным сопротивлением в любой момент времени равна произведению мгновенных значений силы тока i и напряжения и. Следовательно, мгновенная мощность р не является постоянной величиной, как при постоянном токе, а изменяется по кривой (см.

рис. 175,б). Эту кривую можно также получить графически, перемножая ординаты кривых силы тока i и напряжения и при различных углах ?t.

Изменение мощности происходит с двойной частотой ?t по отношению к изменению тока и напряжения, т. е. один период изменения мощности соответствует половине периода изменения тока и напряжения.

Все значения мощности являются положительными. Физически положительное значение мощности означает, что энергия передается от источника электрической энергии к приемнику. Максимальное значение мощности при ?t = 90° и ?t = 270°

Pmax= UтIт= 2UI

Рис. 175. Схема включения в цепь переменного тока активного сопротивления (а), кривые тока i, напряжения и, мощности р (б) и векторная диаграмма (в)

Практически об энергии W, создаваемой электрическим током, судят не по максимальной мощности, а по средней мощности Рср= Р, так как эта энергия может быть выражена как произведение среднего значения мощности Р на время протекания тока:

W = Pt.

Кривая мгновенной мощности симметрична относительно линии АБ, которая соответствует среднему значению мощности Р. Поэтому

P = Pmax/ 2 = UI

Используя формулу (67) закона Ома, активную мощность можно выразить также в виде P = I2R или P=U2/R.

В электротехнике среднюю мощность, потребляемую активным сопротивлением, обычно называют активной мощностью, или просто мощностью, и обозначают буквой Р.

Поверхностный эффект. Следует отметить, что активное сопротивление проводников в цепи переменного тока всегда больше их сопротивления в цепи постоянного тока.

Переменный ток i не протекает равномерно по всему поперечному сечению проводника, как постоянный ток i, а вытесняется на его поверхность (рис. 176, а). Поэтому полезное сечение проводника как бы уменьшается и сопротивление его при переменном токе возрастает.

Это явление носит название поверхностного эффекта. Неравномерное распределение переменного тока по поперечному сечению проводника объясняется действием э. д.

с. самоиндукции, индуцированной в проводнике магнитным полем, которое создается проходящим по проводнику током I. Это магнитное поле действует не только в пространстве, окружающем проводник (внешний поток Ф2), но и внутри самого проводника (внутренний поток Ф2) (рис.

176,б). Поэтому слои проводника, расположенные ближе к его центру, будут охватываться большим магнитным потоком, чем слои, расположенные ближе к его поверхности, и э. д.

с. самоиндукции, индуцированная во внутренних слоях, будет большей, чем во внешних. Поскольку э.

д. с. самоиндукции препятствует изменению

Рис. 176. Схема протекания постоянного I и переменного i токов по проводнику (а) и возникновение поверхностного эффекта (б)

Рис. 177. Схема термообработки деталей токами высокой частоты: 1 — высокочастотный индуктор; 2 — закаливаемая деталь; 3 — разогретый слой

тока, последний будет стремиться пройти там, где э. д.

с. самоиндукции имеет наименьшее значение, т. е.

пройдет преимущественно по поверхностным слоям проводника. В результате этого плотность тока У в поверхностных слоях будет больше, чем во внутренних. Чем больше частота тока, тем больше э.

д. с. самоиндукции индуцируется во внутренних слоях проводника и тем в большей степени ток вытесняется на поверхность.

При частоте 50 Гц увеличение сопротивления медных и алюминиевых проводников при малом их диаметре практически ничтожно, и сопротивление таких проводников в цепях переменного и постоянного тока можно считать одинаковым. Но для медных и алюминиевых проводников диаметром свыше 10 мм, а для стальных проводников при еще меньших диаметрах необходимо при расчетах учитывать влияние поверхностного эффекта на их активное сопротивление.

При токах высокой частоты, принятых в радиотехнике, телевидении и различных высокочастотных установках, с целью лучшего использования металла проводников их обычно изготовляют полыми.

На свойстве переменного тока высокой частоты протекать, главным образом, по поверхности проводников основаны различные методы высокочастотной закалки и термообработки.

Например, при высокочастотной термообработке деталей вихревыми токами (рис. 177) эти токи индуцируются в основном в поверхностном слое металла. Они быстро разогревают поверхностные слои обрабатываемой детали, раньше, чем ее внутренняя часть успеет заметно нагреться за счет теплопроводности металла.

[ads-pc-2]

В электрической цепи переменного токасуществует два вида сопротивлений:активноеи реактивное. Это является существенным отличием от цепей постоянного тока.

Активное сопротивление

При прохождении тока через элементы, имеющие активное сопротивление, потери выделяющейся мощности необратимы. Примером может служить резистор, выделяющееся на нем тепло, обратно в электрическую энергию не превращается. Кроме резистора активным сопротивлением может обладать линии электропередач, соединительные провода, обмотки трансформатора или электродвигателя.

Отличительной чертой элементов имеющих чисто активное сопротивление – это совпадение по фазе тока и напряжения, поэтому вычислить его можно по формуле

Активное сопротивление зависит от физических параметров проводника, таких как материал, площадь сечения, длина, температура.

Реактивное сопротивление

При прохождении переменного тока через реактивные элементы возникает реактивноесопротивление. Оно обусловлено в первую очередь ёмкостями и индуктивностями.

Индуктивностью в цепи переменного тока обладает катушка индуктивности, причём в идеальном случае, активным сопротивлением её обмотки пренебрегают. Реактивное сопротивление катушки переменному току создаётся благодаря её ЭДС самоиндукции. Причем с ростом частоты тока, сопротивление также растёт.

Реактивное сопротивление катушки зависит от частоты тока и индуктивности катушки

Конденсатор обладает реактивным сопротивлением благодаря своей ёмкости. Его сопротивление с увеличением частоты тока уменьшается, что позволяет его активно использовать в электронике в качестве шунта переменной составляющей тока.

Сопротивление конденсатора можно рассчитать по формуле

Треугольник сопротивлений

Цепи переменного тока обладают полным сопротивлением. Полное сопротивление цепи определяется как сумма квадратов активного и реактивного сопротивлений

Графическим изображением этого выражения служит треугольник сопротивлений, который можно получить в результате расчёта последовательной RLC-цепи. Выглядит он следующим образом:На треугольнике видно, что катетами являются активное и реактивное сопротивление, а полной сопротивление гипотенуза.Величина и начальная фаза переменного тока, создаваемого переменным напряжением, зависят не только от величины сопротивлений, образующих электрическую цепь, но и от индуктивности и емкости этой цепи.Активное сопротивление в цепи переменного тока.Строго говоря, любая электрическая цепь обладает, кроме сопротивления, также индуктивностью и емкостью. Если по проводнику проходит ток, то вокруг него возбуждается магнитное поле, т.

е. имеют место явления индуктивности. Ток возникает под действием электрического поля на заряды, следовательно, проводник должен обладать емкостью, так как в диэлектрической среде вокруг него возникает поток смещения.Однако в ряде случаев относительная роль двух из трех параметров  R, L, С в электрической цепи практически незначительна.

Это позволяет рассматривать подобную цепь как обладающую только сопротивлением, или только индуктивностью, или только емкостью.Мы рассмотрим поочередно условия в трех таких простейших цепях переменного тока.В цепи, содержащей только сопротивление г, синусоидальное напряжени u = Um sin ?t источника электроэнергии создает ток:i = u : r = (Um: r ) sin ?tТак как сопротивление r от времени не зависит, то в этой цепи ток совпадает по фазе с напряжением (рис. 1) и изменяется также синусоидально:i = Imsin ?tздесь:Im= Um: rРисунок 1 Кривые мгновенных значений напряжения и тока в цепи,содержащей только сопротивление r.Разделив последнее выражение на , получим формулу закона Омадля действующих значений напряжения и тока:I = U : rКак видно из формулы, этот закон для цепей переменного тока, содержащих только сопротивление r, имеет такой же вид, как и закон Ома для цепи постоянного тока.В цепи переменного тока сопротивление r называется активным сопротивлением. Это сопротивление, в котором электроэнергия преобразуется в другую форму (в теплоту и др.).Оно может существенно отличаться от сопротивления rпри постоянном токе.

Сопротивление для постоянного тока называют омическим, чтобы отличить его от активного сопротивления для переменного тока.Различие между активным и омическим сопротивлениями обуславливается рядом причин. Одна из них – поверхностный эффект, частичное вытеснение переменного тока в поверхностные слои проводника.Чем больше частота переменного тока, тем это вытеснение значительнее. Из-за поверхностного эффекта сопротивлениеrоказывается уже существенно большим, чем вычисленное по формуле:r = ?

(l : S)Поверхностный эффект создается тем, что переменное магнитное поле индуктирует во внешних слоях проводника меньшую ЭДС самоиндукции, чем во внутренней его части.Особенно сильно поверхностный эффект увеличивает активное сопротивление стальных проводов. На активное сопротивление медных и алюминиевых проводов при промышленной частоте поверхностный эффект существенно влияет только при больших сечениях проводов (свыше 25 кв. мм).Кроме поверхностного эффекта, большое увеличение активного сопротивления электрической цепи могут вызывать потери энергии в переменном электромагнитном поле цепи от гистерезиса и вихревых токов.Поделитесь полезной статьей:

Источники:

  • electrosam.ru
  • electrono.ru
  • electroandi.ru
  • fazaa.ru

Активное сопротивление в цепи переменного тока

Электрическая цепь предполагает наличие в своем составе ряда различных компонентов. Конденсаторы и элементы индуктивности включены в схему для получения разного эффекта. Сопротивления присутствуют в виде отдельных элементов-резисторов и резисторов соединения проводов. С физикой влияние компонента схемы на законы протекания электрических явлений было изучено достаточно тщательно и мало отличается от природы протекания электрического тока, будь то под влиянием переменного или постоянного напряжения.

Сопротивление в цепи переменного тока работает так же, как и в режиме постоянного напряжения потока. Другое дело — элементы емкости и индуктивности. Если в подсхеме, в которую включен конденсатор, отсутствует постоянная составляющая тока, то на индуктивность она никак не влияет. Прохождение постоянного тока в индуктивности влияет только на активное сопротивление катушки.

Совершенно иная ситуация в описании электрических процессов в цепи переменного тока.Конденсаторы становятся проводниками, а индукторы (дроссель, трансформатор и т. Д.) Находят индуктивное сопротивление, которое, в свою очередь, играет гораздо более важную роль, а сопротивление часто просто не принимается во внимание.

Но, тем не менее, необходимость точных расчетов требует учитывать эту составляющую. Чтобы начать понимать, как активное сопротивление катушки сочетается с индуктивным, следует в целом рассматривать устройство как стандартный индуктор.

Являясь составной частью электрической цепи, данное устройство не что иное, как биполярный элемент множества электрических, электромеханических и электронных систем и устройств.В качестве основного параметра используется значение самоиндукции. Это, в свою очередь, зависит только от геометрических размеров и материалов изготовления. На величину индуктивности не влияет ни сила тока, ни напряжение. Использование индукторов в фильтрах для подавления помех, их использование позволяет добиться сглаживания, также катушка может накапливать энергию, что широко используется при проектировании резонансных цепей.

Чтобы проанализировать, какое влияние оказывает сопротивление работе дроссельной заслонки, вы должны рассмотреть цепь переменного тока с одним элементом, которым является индуктор.Рассчитывая активное и индуктивное сопротивление, казалось бы, самый простой способ вычислить общие значения — это сумма их модулей.

Но с такими количествами дело обстоит не так просто, как могло бы показаться. Опуская теоретическое обоснование, опишите использование на практике метода суммирования активного и индуктивного сопротивлений.

Чтобы найти общий балл, постройте прямоугольный треугольник. Одна сторона — это сопротивление, а другая — индуктивная. Гипотенуза будет равна сопротивлению цепи, которое согласно определению равно квадратному корню из суммы квадратов активного и индуктивного сопротивлений.

Выполненные таким образом расчеты дают более точную информацию о процессах, происходящих в цепи переменного тока, элементами которой являются индукторы. В формуле закона Ома есть возможность использовать величину импеданса. В будущем следует отметить, что значительное сопротивление по сравнению с индуктивным может влиять на фазовый сдвиг между током и напряжением. Поэтому при изготовлении катушек индуктивности широко используется конструкция сердечника, которая дает значительные преимущества по индуктивному сопротивлению.

Активное сопротивление в цепи переменного тока

Электрическая схема предполагает наличие в ее составе ряда различных компонентов. Конденсаторы и индукторы включены в схему, чтобы произвести самый разнообразный эффект. Активное сопротивление представлено отдельными резистивными элементами и сопротивлением соединительных проводов. Физика влияния этого компонента цепи на законы протекания электрических явлений изучена довольно тщательно и мало отличается от природы протекания электрического тока, будь то под действием переменного или постоянного напряжения.

Активное сопротивление в цепи переменного тока работает так же, как и в режиме постоянного напряжения. Другое дело элементы емкости и индуктивности. Если на участке цепи, к которому подключен конденсатор, постоянная составляющая тока полностью отсутствует, то катушки индуктивности не оказывают на нее никакого влияния. На протекание постоянного тока в варианте с индуктивностью влияет только активное сопротивление катушки.

Совершенно иная ситуация возникает при описании электрических процессов в цепи переменного тока.Конденсаторы становятся проводниками, а индукторы (дроссели, обмотки трансформатора и т. Д.) Приобретают индуктивное сопротивление, которое, в свою очередь, играет гораздо более важную роль, а активное сопротивление часто просто не учитывается.

Но, тем не менее, необходимость точных расчетов требует учета этой составляющей. Для начала, чтобы понять, как активное сопротивление катушки сочетается с индуктивным, необходимо в общих чертах обрисовать устройство стандартной катушки индуктивности.

Являясь составной частью электрической схемы, это устройство является не чем иным, как биполярным элементом самых разнообразных электрических, электромеханических и электронных систем и устройств. В качестве основного параметра он использует значение собственной индуктивности. То же в свою очередь зависит только от геометрических размеров и материалов изготовления. На величину индуктивности не влияют ни ток, ни напряжение. В фильтрах нашли применение дроссель для подавления помех, их использование позволяет добиться сглаживания пульсаций, а катушка может накапливать энергию, что широко используется при построении колебательных цепей.

Чтобы проанализировать, какое влияние активное сопротивление оказывает на работу дроссельной заслонки, рассмотрим цепь переменного тока с одним элементом, которым является индуктор. Казалось бы, самый простой способ вычислить суммарное значение активного и индуктивного сопротивления — сложить их по модулю.

Но с этими количествами дело обстоит не так просто, как может показаться. Опуская теоретическое обоснование, опишем использование на практике метода суммирования активного и индуктивного сопротивления.

Чтобы найти итоговый показатель, строим прямоугольный треугольник. Одна из его ножек — активное сопротивление, а другая — индуктивное сопротивление. Гипотенуза будет равна общему сопротивлению цепи, которое согласно определению будет равно квадратному корню из суммы квадратов активного и индуктивного сопротивлений.

Проведенные таким образом расчеты несут более точную информацию о процессах, происходящих в цепи переменного тока, элементами которой являются катушки индуктивности.В формуле закона Ома мы можем использовать значение импеданса. В будущем следует отметить, что значительное активное сопротивление по сравнению с индуктивным может влиять на фазовый сдвиг между током и напряжением. Именно поэтому конструкция с сердечником широко используется при производстве дросселей, что дает значительные преимущества индуктивному сопротивлению.

Мощность в цепи переменного тока — Университетская физика, том 2

Цели обучения

К концу раздела вы сможете:

  • Опишите, как можно записать среднюю мощность в цепи переменного тока через пиковый ток и напряжение, а также среднеквадратичные значения тока и напряжения.
  • Определите соотношение между фазовым углом тока и напряжения и средней мощностью, известное как коэффициент мощности.

Элемент схемы рассеивает или вырабатывает мощность в зависимости от того, где I — ток через элемент, а В, — напряжение на нем.Поскольку ток и напряжение в цепи переменного тока зависят от времени, мгновенная мощность также зависит от времени. График p ( t ) для различных элементов схемы показан на (Рисунок). Для резистора i ( t ) и v ( t ) синфазны и поэтому всегда имеют один и тот же знак (см. (Рисунок)). Для конденсатора или катушки индуктивности относительные знаки i ( t ) и v ( t ) меняются в течение цикла из-за разницы фаз (см. (Рисунок) и (Рисунок)).Следовательно, p ( t ) в одни моменты положительный, а в другие — отрицательный, что указывает на то, что емкостные и индуктивные элементы вырабатывают энергию в одни моменты и поглощают ее в другие.

Поскольку мгновенная мощность изменяется как по величине, так и по знаку в течение цикла, она редко имеет какое-либо практическое значение. То, что нас почти всегда интересует, — это усредненная по времени мощность, которую мы называем средней мощностью. Он определяется средним по времени мгновенной мощностью за один цикл:

где — период колебаний.С заменами и этот интеграл становится

Используя тригонометрическое соотношение, получаем

Вычисление этих двух интегралов дает

и

Следовательно, средняя мощность, связанная с элементом схемы, равна

В технических приложениях известен как коэффициент мощности, который представляет собой величину, на которую мощность, передаваемая в цепи, меньше теоретического максимума цепи из-за того, что напряжение и ток не совпадают по фазе.Для резистора, поэтому средняя рассеиваемая мощность составляет

Сравнение p ( t ) и показано на (Рисунок) (d). Чтобы выглядеть как его аналог постоянного тока, мы используем среднеквадратичные значения тока и напряжения. По определению это

где

С получаем

Тогда мы можем написать для средней мощности, рассеиваемой резистором,

Это уравнение дополнительно подчеркивает, почему при обсуждении выбирается среднеквадратичное значение, а не пиковые значения.Оба уравнения для средней мощности верны для (рисунок), но среднеквадратичные значения в формуле дают более четкое представление, поэтому дополнительный коэффициент 1/2 не требуется.

Переменные напряжения и токи обычно описываются их действующими значениями. Например, напряжение 110 В от бытовой розетки является среднеквадратичным значением. Амплитуда этого источника равна. Поскольку большинство измерителей переменного тока откалиброваны по среднеквадратичным значениям, типичный вольтметр переменного тока, помещенный в бытовую розетку, будет показывать 110 В.

Для конденсатора и катушки индуктивности соответственно. Поскольку мы находим из (Рисунок), что средняя мощность, рассеиваемая любым из этих элементов, равна Конденсаторам, а катушки индуктивности поглощают энергию из цепи в течение одного полупериода, а затем разряжают ее обратно в цепь в течение другого полупериода. Это поведение проиллюстрировано на графиках (Рисунок), (b) и (c), которые показывают, что p ( t) колеблются синусоидально около нуля.

Фазовый угол генератора переменного тока может иметь любое значение.Если генератор выдает мощность; если он поглощает энергию. В терминах среднеквадратичных значений средняя мощность генератора переменного тока записывается как

.

Для генератора в цепи RLC ,

и

Отсюда средняя мощность генератора

Это также можно записать как

, который означает, что мощность, производимая генератором, рассеивается в резисторе. Как мы видим, закон Ома для среднеквадратичного значения переменного тока находится делением среднеквадратичного напряжения на импеданс.

Проверьте свое понимание Вольтметр переменного тока, подключенный к клеммам генератора переменного тока 45 Гц, показывает 7,07 В. Напишите выражение для ЭДС генератора.

Проверьте свое понимание Покажите, что действующие значения напряжения на резисторе, конденсаторе и катушке индуктивности в цепи переменного тока, где среднеквадратичный ток выражается соответственно. Определите эти значения для компонентов цепи RLC (рисунок).

Сводка

  • Средняя мощность переменного тока определяется умножением среднеквадратичных значений тока и напряжения.
  • Закон Ома для среднеквадратичного значения переменного тока находится делением среднеквадратичного напряжения на полное сопротивление.
  • В цепи переменного тока существует фазовый угол между напряжением источника и током, который можно найти, разделив сопротивление на полное сопротивление.
  • Средняя мощность, подаваемая в цепь RLC , зависит от фазового угла.
  • Коэффициент мощности находится в диапазоне от –1 до 1.

Концептуальные вопросы

При каком значении фазового угла между выходным напряжением источника переменного тока и током средняя выходная мощность источника является максимальной?

Обсудите разницу между средней мощностью и мгновенной мощностью.

Мгновенная мощность — это мощность в данный момент. Средняя мощность — это мощность, усредненная за цикл или количество циклов.

Средний переменный ток, подаваемый в цепь, равен нулю.Несмотря на это, мощность в цепи рассеивается. Объяснять.

Может ли мгновенная выходная мощность источника переменного тока быть отрицательной? Может ли средняя выходная мощность быть отрицательной?

Мгновенная мощность может быть отрицательной, но выходная мощность не может быть отрицательной.

Номинальная мощность резистора, используемого в цепях переменного тока, относится к максимальной средней мощности, рассеиваемой в резисторе. Как это соотносится с максимальной мгновенной мощностью, рассеиваемой на резисторе?

Глоссарий

средняя мощность
среднее по времени мгновенной мощности за один цикл
Коэффициент мощности
величина, на которую мощность, передаваемая в цепи, меньше теоретического максимума цепи из-за того, что напряжение и ток не совпадают по фазе

Что такое активная реактивная и импедансная.Реактивное сопротивление XL и XC. Полное сопротивление цепи с последовательным соединением активного и реактивного сопротивления

Итак, индукторы и конденсаторы препятствуют протеканию переменного тока. Такое сопротивление переменного тока называется реактивным сопротивлением X и измеряется в Омах. Реактивное сопротивление Зависит как от значения индуктивности и емкости, так и от частоты сигнала.

Катушка индуктивности

имеет индуктивное реактивное сопротивление ВЛ равное

, где F — частота в Герцах, A L — индуктивность в Генри.
Поскольку ω = 2πf, можно написать XL = Ωl. Например, реактивное сопротивление катушки с индуктивностью 10 мПН, подаваемой с частотой 1 кГц, равно

.

XL = 2π * 1 * 103 * 10 * 10-3 = 62,8 Ом.

Реактивное сопротивление катушки индуктивности увеличивается с увеличением частоты сигнала (рис. 4.26).
Конденсатор имеет емкостное сопротивление XC, равное

.

, где C — контейнер в Фарадах. Например, реактивное сопротивление конденсатора емкостью 1 мкФ, которое подается с частотой 10 кГц, равно

.


Рис.4.26. Зависимость индукционного риса. 4.27.
сопротивление от частоты.


Рис. 4.28. Емкостная векторная сумма (XC)

и индуктивное (XL) сопротивление.

Рис. 4.29.
(а) катушка индуктивности, последовательно соединенная с резистором R.
(б) векторное представление R, XL и их векторная сумма z

Реактивное сопротивление конденсатора уменьшается с увеличением частоты сигнала (рис. 4.27).
Результирующее сопротивление цепи включает емкостное сопротивление XC и индуктивное сопротивление XL, равное векторной сумме XC и XL.Векторы XC и XL, как видно на рис. 4.28 (b) находятся в противофазе, то есть разность фаз между ними составляет 1800. Следовательно, результирующее сопротивление просто равно разнице между XC и XL. Например, пусть xl = 100 Ом, а xc = 70 Ом. Тогда результирующее реактивное сопротивление x = 100 — 70 = 30 Ом и будет индуктивным, так как XL больше XC.

Импеданс
Результирующее сопротивление цепи, содержащее как активное, так и реактивное (индуктивное или емкостное) сопротивление, является названием импеданса или общего сопротивления цепи.
Импеданс Z — это вектор величины реактивного сопротивления и активного сопротивления R.
Рассмотрим, например, схему, показанную на рис. 4.29. Он включает в себя индуктивное сопротивление XL, подключенное последовательно с резистором R. Как видно из рис. 4.29 (б), вектор XL опережает вектор R на 90 °. Импеданс равен

Если xl = 400 Ом и r = 300 Ом, то z = 500 Ом.

Реактивное сопротивление — электрическое сопротивление Переменный ток, возникающий в результате передачи энергии магнитным полем в индукторах или электрическим полем в конденсаторах.

Элементы, обладающие реактивным сопротивлением, называются реактивными.

Реактивное сопротивление катушки индуктивности.

При протекании переменного тока I. В катушке магнитное поле создает в ее витках EDC, что предотвращает изменение тока.
При увеличении тока ЭДС отрицательная и препятствует увеличению тока, при уменьшении — положительная и предотвращает его уменьшение, таким образом сопротивление изменению тока на протяжении всего периода.

В результате созданного противодействия на выводах индуктивности индуктивности в противофазе формируются напряжение U. подавляющая ЭДС равной амплитуде и противоположного знака.

При прохождении тока через ноль амплитуда ЭДС достигает максимального значения, что формирует несоответствие текущего времени и напряжения в 1/4 периода.

Если подать на выводы катушки индуктивности напряжение U. , то ток не может начаться мгновенно из-за противодействия EDC, равного -U. Следовательно, ток в индуктивности всегда будет отставать от напряжения на угол 90 °. Сдвиг с запаздывающим током называется положительным.

Запишем выражение мгновенное значение напряжения мкА. На основе ЭДС ( ε ), которая пропорциональна индуктивности л. и изменениям тока: u = -ε = L (di / dt) .
Отсюда мы выражаем синусоидальный ток.

Интегральная функция sIN (T) будет —SS (T) или равна ее функции sIN (T-π / 2) .
Дифференциал dt. Функции sin (ωt) выйдет из-под знака интегрального множителя 1 / ω .
В результате получаем мгновенное выражение тока со смещением от влияния напряжения π / 2. (90 °).
Для среднеквадратичных значений U. и I. В этом случае вы можете записывать.

В итоге имеем зависимость синусоидального тока от напряжения по закону Ома, где в знаменателе вместо Р. выражение ωL , которое является реактивным сопротивлением:

Реактивное сопротивление индукторов называется индуктивным.

Реактивное сопротивление конденсатора.

Электрический ток в конденсаторе — это часть или совокупность процессов его заряда и разряда — накопления и возврата энергии электрическим полем между его пластинами.

В цепи переменного тока конденсатор будет заряжаться до определенного максимального значения, пока ток не изменит направление на противоположное.Следовательно, в моменты амплитудного значения напряжения на конденсаторе ток в нем будет нулевым. Таким образом, напряжение на конденсаторе и ток всегда будут иметь несоответствие во времени в четверть периода.

В результате ток в цепи будет ограничен падением напряжения на конденсаторе, что создает реактивное сопротивление переменному току, обратно пропорциональную скорость изменения тока (частоты) и емкость конденсатора.

При подаче на конденсатор напряжения U., ток мгновенно начнется с максимального значения, далее уменьшаясь до нуля. В это время напряжение на его выводах будет расти от нуля до максимального. Следовательно, напряжение на пластинах фазового конденсатора отстает от тока на угол 90 °. Такой фазовый сдвиг называется отрицательным.

Ток в конденсаторе является производной функции его заряда i = dq / dt = c (du / dt) .
Получено из sIN (T) будет cOS (T) или аналогичная функция sIN (T + π / 2) .
Тогда для синусоидального напряжения u = u amp sin (ωt) Запишем выражение мгновенное значение тока следующим образом:

i = u amp ωcsin (ωt + π / 2) .

Отсюда Выразите соотношение среднеквадратичных значений.

Закон Ома предполагает, что 1 / Омc. Нет ничего, кроме реактивного сопротивления для синусоидального тока.

Сопротивление, которое оказывает проводник, пропускающий по нему переменный ток, называется активным сопротивлением .

Если какой-либо потребитель не содержит индуктивности и резервуаров (лампа накаливания, нагревательное устройство), он также будет для переменного тока в качестве активного сопротивления.

Активное сопротивление зависит от частоты переменного тока, которая увеличивается с ее увеличением.

Однако многие потребители обладают индуктивными и емкостными свойствами при пропускании через них переменного тока. К таким потребителям относятся трансформаторы, дроссели, электромагниты, конденсаторы, разного рода провода и многие другие.

При прохождении через них необходимо учитывать не только активное, но и реактивное сопротивление за счет наличия у потребителя индуктивных и емкостных свойств у него.

Активное сопротивление Определяет фактическую часть импеданса:

Где — импеданс, — величина активного сопротивления, величина реактивного сопротивления, мнимая единица.

Активное сопротивление — сопротивление электрической цепи или ее участка вследствие необратимых преобразований электрической энергии в другие виды энергии (в тепловую энергию)

Реактивное сопротивление — электрическое сопротивление за счет передачи энергии переменным током, электрическим или магнитным полем (и обратно).

Величину реактивного сопротивления можно выразить через значения индуктивного и емкостного сопротивления:

Величина полного реактивного сопротивления

Индуктивное сопротивление () Возникновение ЭДС самоиндукции в элементе электрической цепи.

Емкость ().

Здесь — циклическая частота

Импеданс Цепи переменного тока:

z =. R 2 + x 2 = R 2 + (X L -X C) 2

Билет № 12.

1. 1) Согласование генератора с нагрузкой — , обеспечивающее необходимое значение активного эквивалентного сопротивления нагрузки генераторной лампы R e со всеми возможными значениями входного импеданса антенного фидера, зависящего от его волновое сопротивление и коэффициент поперечной волны ( CBW)

Координация (в электронике) сводится к правильному выбору сопротивлений генератора (источника), линий передачи и приемника (нагрузки).Идеального согласования (в электронике) между линией и нагрузкой можно добиться при равенстве волнового сопротивления линии R полному сопротивлению нагрузки ZH = RH + J HN, либо при Rh = R и Xh = 0, где Rh-активная часть импеданса, Xh его реактивная часть. В этом случае линия передачи устанавливает режим резиновых волн и характеризующий их коэффициент стоячей волны (CWS) равен 1. Для линии При ничтожно малых потерях электрической энергии согласование и, как следствие, наиболее эффективная передача. энергии от генератора в нагрузке достигается при условии, что полные сопротивления генератора Zr и нагрузки ZH комплексно сопряжены, то есть Zr = z * H, или RR = R = RH \ u003d XR — XH.При этом реактивное сопротивление цепи равно нулю, и соблюдаются условия резонанса, способствующие повышению эффективности радиотехнических систем (улучшается использование частотных диапазонов, повышается помехозащищенность, уменьшаются частотные искажения радиосигналов и т. Д.). Оценка качества согласования (в электронике) производится путем измерения коэффициента отражения и КСВ. Практически согласование (в электронике) считается оптимальным, если рабочая полоса частоты CWW не превышает 1.2-1.3 (в средствах измерений 1.05). В некоторых случаях косвенные индикаторы (в электронике) могут служить реакцией параметров генератора (частота, мощность, уровень шума) на изменение нагрузки, наличие электрических пробоев в линии, нагрев отдельных участков линии. .

При таком режиме работы в приемнике максимальная мощность равна половине мощности источника. В этом случае K.P.D. = 0,5. Этот режим используется в измерительных схемах, устройствах связи.

При передаче больших мощностей, например по высоковольтным линиям электропередачи, работа в согласованном режиме обычно недопустима.

Цепь переменного электрического тока включает в себя активные (содержащие внутренние источники энергии) и пассивные элементы (потребители энергии). К пассивным элементам относятся резисторы и жиклеры.

Типы пассивных элементов

В электротехнике рассматриваются два типа резисторов: активное и реактивное сопротивление.Активные — усовершенствованные устройства, в которых энергия электрического тока преобразуется в тепловую. В физике обозначается символом R. Единица измерения — ОМ.

Эту формулу можно использовать для расчета мгновенных значений тока и напряжения, максимальных или действительных.

Струйные устройства не рассеивают энергию, а накапливают ее. К ним относятся:

Реактивное сопротивление обозначается символом H. Единица измерения — Ом.

Индуктор

Это проводник, выполненный в виде спирали, винта или спиралей.Из-за большой инерции устройство используется в схемах, которые используются для уменьшения пульсаций в цепях переменного тока и колебательных цепях, для создания магнитного поля и т. Д. Если у него большая длина при малом диаметре, то катушка называется соленоидом.

Для расчета падения напряжения ( U. ) на концах катушки используйте формулу:

U = -l · di / dt, где:

  • L — индуктивность устройства, измеряется в GN (Генри),
  • DI — Изменение силы тока (измеряется в амперах) в течение временного интервала DT (измеряется в секундах).

Внимание! При любом изменении тока в проводнике возникают ЭМИ самоиндукции, которые предотвращают это изменение.

В результате в катушке возникает сопротивление, которое называют индуктивным.

В электротехнике обозначается x L. И вычисляется по формуле:

где W — угловая частота, измеряемая в рад / с.

Угловая частота является характеристикой гармоничного колебания.Связан с частотой F (количество полных колебаний в секунду). Частота измеряется в колебаниях в секунду (1 / с):

w = 2 · p · f.

Если на схеме несколько катушек, то при их использовании последовательное соединение General H. L. для всей системы будет равно:

XL = XL1 + XL2 + …

В случае параллельного соединения:

1 / XL = 1 / XL1 + 1 / XL2 +…

Закон Ома для такого соединения имеет вид:

, где UL — падение напряжения.

Помимо индуктивного, устройство имеет оба активных R.

Электрическое сопротивление в данном случае составляет:

Емкостный элемент

В проводниках и обмотке катушки, помимо индуктивного и активного сопротивлений, присутствует еще и емкостное, что связано с наличием в этих устройствах бака. Помимо резистора и катушки в схему может быть включен конденсатор, который состоит из двух металлических пластин, между которыми размещен диэлектрический слой.

Для информации. Электрический ток течет из-за того, что устройство проходит и процессы разряда проходят.

При максимальном заряде на пластинах прибора:

Благодаря тому, что резистивное устройство может накапливать энергию, оно используется в устройствах, стабилизирующих напряжение в цепи.

Способность накапливать заряд характеризуется емкостью.

Реактивное сопротивление конденсатора (CC) можно рассчитать по формуле:

Xc = 1 / (w · c), где:

  1. w — угловая частота,
  2. C — емкость конденсатора. конденсатор.

Единица измерения емкости — Ф (Фарадей).

Учитывая, что угловая частота связана с циклической частотой, расчет значения реактивного сопротивления конденсатора можно произвести по формуле:

Xc = 1 / (2 · p · f · c).

Если в схему подключено несколько устройств, то всего X. ИЗ Систем будет равно:

Xc = xc1 + xc2 + …

Если соединение объектов параллельное, то:

1 / xc = 1 / xc1 + 1 / xc2 + …

Закон Ома для этого случая записывается следующим образом:

где Us падение напряжения на конденсаторе.

Расчет цепи

При последовательном подключении I. = конст. Любая точка и, согласно закону Ома, ее можно рассчитать по формуле:

где Z — электрическое сопротивление.

Напряжение на устройствах рассчитывается следующим образом:

Ur = i · r, ul = i · xl, uc = i · xc.

Вектор индуктивной составляющей напряжения направлен в противоположном направлении от вектора емкостной составляющей, поэтому:

следовательно, согласно расчетам:

Внимание! Для расчета значения импеданса можно использовать «треугольник сопротивлений», в котором гипотенуза равна z, а по категориям — значения X и R.

Если конденсатор и катушка индуктивности соединены в цепь, то, согласно Пифагору, теорема гипотенузы ( Z. ) будет равна:

As X . = XL ХС. , затем:

При решении электрических задач импеданс часто записывается в виде комплексного числа, в котором фактическая часть соответствует значению активного компонента, а мнимая часть является реактивной. Таким образом, выражение для импеданса в целом имеет вид:

, где I — мнимая единица.

Для онлайн-расчета реактивного сопротивления вы можете использовать программу — калькулятор, которую можно найти в Интернете. Таких сервисов очень много, поэтому выбрать удобный для вас калькулятор не составит труда.

Благодаря этому Интернет-сервису вы можете быстро выполнить требуемый расчет.

Видео

Одной из основных проблем в сети переменного напряжения является наличие реактивной мощности.Расходуется только на тепловые потери. Источником реактивной энергии являются электрические приводы L и C. Я не буду очень глубоко рассматривать этот вопрос. Предлагаю рассмотреть этот вопрос на примере простых элементов цепи — индуктивности и емкостей.

Индуктивный элемент L.

Индуктивный элемент (Рассмотрим на примере катушки индуктивности) витки изолированных между собой проводов. При протекании тока катушка намагничивается. Если вы измените полярность источника, катушка начнет отдавать накопленную энергию обратно, пытаясь поддерживать текущее значение в цепи.Следовательно, когда через него протекает переменная составляющая, энергия, накопленная во время прохождения положительного полупериода, не успевает рассеяться и будет препятствовать прохождению отрицательного полупериода. В результате отрицательный полупериод должен будет расплатиться с накопленной катушкой энергии. В результате напряжение (U) будет опережать ток (І) на некоторый угол φ. Ниже представлен результат модельных работ на L-R Нагрузка L = 1 * 10 -3 Гн, R = 0,5 Ом. Усть = 250 В, частота F = 50 Гц.

φ — разность фаз между U и I.

Реактивное сопротивление обозначается буквой X, общее z, активное R.

Для индуктивности:

Где ω — циклическая частота

L — индуктивность катушки;

Вывод: чем выше индуктивность L или частота, тем больше сопротивление катушки переменному току.

Емкостный элемент

Емкостный элемент (рассмотрим на примере конденсатора) — двухстоечный с переменным или постоянным значением емкости.Конденсатор — привод электрических зарядов. Если подключить к источнику питания, он заряжается. Если к нему приложен источник с переменной составляющей, он будет заряжаться, когда через него пройдет положительный полупериод. Когда направление половинной цели должно измениться на отрицательное значение, конденсатор начнет перезаряжаться, то есть энергия, которая в нем накопилась, начнет противодействовать перезарядке. В результате мы получим напряжение на конденсаторе напротив источника. В результате он будет обнаружен u для некоторого угла φ.Ниже представлен результат модельных работ на C-R Нагрузка C = 900 * 10-6 Fa, R = 0,5 Ом, Uст = 250 В, частота F = 50 Гц.


Рисунок 2. Рабочий источник на ПДУ Нагрузка

Для емкости:

Где ω — циклическая частота

— частота питающего напряжения, Гц;

С — емкость конденсатора;

Вывод: Чем выше емкость C или частота, тем меньше сопротивление переменному току.

Сравнение влияния реактивного сопротивления на активную мощность сети

На рисунках 1 и 2 видно, что фазовый сдвиг на рисунках не одинаковый.Выход — чем больше в полном сопротивлении Z будет влияние X L или X C. Чем больше будет разница фаз U и I.

Угол сдвига между током и напряжением называется φ.

Нефазная реактивная мощность:

Трехфазный:

U F, I F — Фазный ток и напряжение

Вывод: реактивная мощность — не оказывает полезного воздействия.

Она «перегоняет» по сети греющие кабели и увеличивает потери.На крупных промышленных предприятиях это особенно заметно из-за наличия электроприводов и другие крупные потребители. Этот вопрос очень актуален для энергосбережения и модернизации производства. Поэтому на пром. На предприятиях установлены компенсаторы реактивной мощности. Они могут быть разных типов и кроме компенсации выполнять роль фильтров. С помощью компенсаторов стараются поддерживать баланс реактивной мощности, чтобы минимизировать ее влияние на сеть и довести угол φ до нуля.

Для этого необходимо максимально увеличить количество (L, C) элементов в сети.

РЕАКТИВНАЯ МОЩНОСТЬ — прикладное промышленное электричество

Рисунок 6.1 Чисто резистивная цепь переменного тока: напряжение и ток резистора синфазны.

Если бы мы изобразили ток и напряжение для очень простой цепи переменного тока, состоящей из источника и резистора (рисунок выше), это выглядело бы примерно так: (рисунок ниже)

Рисунок 6.2 Напряжение и ток «синфазны» для резистивной цепи.

Поскольку резистор просто и напрямую сопротивляется протеканию тока в любое время, форма волны падения напряжения на резисторе точно совпадает по фазе с формой волны тока через него. Мы можем посмотреть в любой момент времени вдоль горизонтальной оси графика и сравнить эти значения тока и напряжения друг с другом (любой «снимок», показывающий значения волны, упоминается как мгновенных значений , что означает значения при этом моментов времени времени).Когда мгновенное значение тока равно нулю, мгновенное напряжение на резисторе также равно нулю. Аналогичным образом, в момент времени, когда ток через резистор находится на своем положительном пике, напряжение на резисторе также находится на своем положительном пике, и так далее. В любой момент времени на волнах закон Ома справедлив для мгновенных значений напряжения и тока.

Мы также можем рассчитать мощность, рассеиваемую этим резистором, и нанести эти значения на тот же график: (рисунок ниже)

Рисунок 6.3 Мгновенная мощность переменного тока в чисто резистивной цепи всегда положительная.

Резисторы и катушки индуктивности

Катушки индуктивности ведут себя иначе, чем резисторы. В то время как резисторы просто препятствуют прохождению тока через них (снижая напряжение, прямо пропорциональное току), индукторы противодействуют изменениям тока через них, понижая напряжение, прямо пропорциональное скорости изменения тока. В соответствии с законом Ленца , это индуцированное напряжение всегда имеет такую ​​полярность, чтобы поддерживать ток на его текущем значении.То есть, если ток увеличивается по величине, индуцированное напряжение будет «противодействовать» протеканию тока; если ток уменьшается, полярность изменится на противоположную и «подтолкнет» ток, чтобы противодействовать уменьшению. Это противодействие текущему изменению называется реактивным сопротивлением , а не сопротивлением. Выражаясь математически, соотношение между падением напряжения на катушке индуктивности и скоростью изменения тока через катушку индуктивности выглядит следующим образом:

[латекс] e = L \ frac {d_i} {d_t} [/ латекс]

Переменный ток в простой индуктивной цепи

Выражение di / dt получено из расчетов, означающее скорость изменения мгновенного тока (i) во времени в амперах в секунду.Индуктивность (L) измеряется в Генри, а мгновенное напряжение (е), конечно, выражается в вольтах. Иногда вы можете встретить скорость мгновенного напряжения, выраженную как «v» вместо «e» (v = L di / dt), но это означает то же самое. Чтобы показать, что происходит с переменным током, давайте проанализируем простую цепь индуктивности:

Рисунок 6.4 Чистая индуктивная цепь: ток индуктора отстает от напряжения индуктора на 90 °.

Если бы мы изобразили ток и напряжение для этой очень простой схемы, это выглядело бы примерно так:

Рисунок 6.5 Чистая индуктивная цепь, формы сигналов.

Помните, что падение напряжения на катушке индуктивности является реакцией на изменение тока через нее на . Следовательно, мгновенное напряжение равно нулю всякий раз, когда мгновенный ток достигает пика (нулевое изменение или наклон уровня на синусоидальной волне тока), а мгновенное напряжение находится на пике везде, где мгновенный ток имеет максимальное изменение (точки самый крутой наклон на текущей волне, где она пересекает нулевую линию).Это приводит к появлению волны напряжения, сдвинутой по фазе на 90 ° с волной тока. Глядя на график, кажется, что волна напряжения имеет «фору» по сравнению с волной тока; напряжение «опережает» ток, а ток «отстает» от напряжения.

Ток отстает от напряжения на 90 ° в чисто индуктивной цепи.

Все становится еще интереснее, когда мы строим график мощности для этой схемы:

Рисунок 6.6 В чисто индуктивной цепи мгновенная мощность может быть положительной или отрицательной.

Поскольку мгновенная мощность является произведением мгновенного напряжения и мгновенного тока (p = ie), мощность равна нулю, когда мгновенный ток или напряжение равно нулю. Если мгновенный ток и напряжение положительные (над линией), мощность положительная. Как и в примере с резистором, мощность также положительна, когда мгновенные ток и напряжение отрицательны (ниже линии). Однако, поскольку волны тока и напряжения сдвинуты по фазе на 90 °, бывают случаи, когда одна из них положительна, а другая — отрицательна, что приводит к одинаково частым возникновению отрицательной мгновенной мощности .

Что такое отрицательная сила?

Но что означает отрицательная мощность ? Это означает, что катушка индуктивности возвращает мощность в цепь, в то время как положительная мощность означает, что она поглощает мощность из цепи. Поскольку положительные и отрицательные циклы мощности равны по величине и продолжительности с течением времени, индуктор возвращает обратно в цепь столько же мощности, сколько потребляет в течение полного цикла. В практическом смысле это означает, что реактивное сопротивление катушки индуктивности рассеивает нулевую полезную энергию, в отличие от сопротивления резистора, который рассеивает энергию в виде тепла.Имейте в виду, это только для идеальных катушек индуктивности, у которых нет сопротивления провода.

Реактивное сопротивление в зависимости от сопротивления

Противодействие катушки индуктивности изменению тока приводит к противодействию переменному току в целом, который по определению всегда изменяется по мгновенной величине и направлению. Это противодействие переменному току аналогично сопротивлению, но отличается тем, что всегда приводит к сдвигу фаз между током и напряжением и рассеивает нулевую мощность. Из-за различий он имеет другое название: реактивное сопротивление .Реактивное сопротивление по переменному току выражается в омах, как и сопротивление, за исключением того, что его математический символ — X вместо R. Чтобы быть конкретным, реактивное сопротивление, связанное с катушкой индуктивности, обычно обозначается заглавной буквой X с буквой L в качестве нижнего индекса, например это: X L .

Так как индуктивности падают напряжение пропорционально скорости изменения тока, они будут падать больше напряжения для более быстро изменяющихся токов и меньше напряжения для более медленно меняющихся токов. Это означает, что реактивное сопротивление в Ом для любой катушки индуктивности прямо пропорционально частоте переменного тока.Точная формула для определения реактивного сопротивления выглядит следующим образом:

[латекс] X_L = 2πfL [/ латекс]

Если мы подвергнем индуктор 10 мГн воздействию частот 60, 120 и 2500 Гц, он проявит реактивные сопротивления, указанные в таблице ниже.

Реактивное сопротивление индуктора 10 мГн:

Таблица 6.1 Реактивное сопротивление индуктора 10 мГн
Частота (Герцы) Реактивное сопротивление (Ом)
60 3.7699
120 7,5398
2500 157.0796

В уравнении реактивного сопротивления термин «2πf» (все в правой части, кроме L) имеет особое значение. Это количество радиан в секунду, на которое «вращается» переменный ток, если вы представите себе один цикл переменного тока, представляющий вращение полного круга. радиан — это единица измерения угла: в одном полном круге 2π радиана, так же как в полном круге 360 °.Если генератор переменного тока является двухполюсным, он будет производить один цикл на каждый полный оборот вала, что составляет каждые 2π радиан или 360 °. Если эту константу 2π умножить на частоту в герцах (циклов в секунду), результатом будет число в радианах в секунду, известное как угловая скорость системы переменного тока.

Угловая скорость в системах переменного тока

Угловая скорость может быть представлена ​​выражением 2πf или ее собственным символом, строчной греческой буквой омега, которая похожа на нашу строчную римскую букву «w»: ω.Таким образом, формула реактивного сопротивления X L = 2πfL также может быть записана как X L = ωL.

Следует понимать, что эта «угловая скорость» является выражением того, насколько быстро колеблются колебания переменного тока, полный цикл равен 2π радиан. Это не обязательно отражает фактическую скорость вала генератора переменного тока. Если генератор имеет более двух полюсов, угловая скорость будет кратной скорости вала. По этой причине ω иногда выражается в единицах электрических радиан в секунду, а не в (простых) радианах в секунду, чтобы отличить его от механического движения.

Как бы мы ни выразили угловую скорость системы, очевидно, что она прямо пропорциональна реактивному сопротивлению в катушке индуктивности. По мере увеличения частоты (или скорости вала генератора переменного тока) в системе переменного тока катушка индуктивности будет оказывать большее сопротивление прохождению тока, и наоборот. Переменный ток в простой индуктивной цепи равен напряжению (в вольтах), деленному на индуктивное реактивное сопротивление (в омах), точно так же, как переменный или постоянный ток в простой резистивной цепи равен напряжению (в вольтах), деленному на сопротивление (в Ом).Пример схемы показан здесь:

Индуктивное сопротивление

(Индуктивное сопротивление индуктора 10 мГн при 60 Гц)

[латекс] X_L = 3,7600 Ом [/ латекс]

[латекс] I_ {X_ {L}} = \ frac {E} {X} [/ latex]

[латекс] = \ frac {10 В} {3,7600 Ом} [/ латекс]

[латекс] \ mathbf {= 2.6526A} [/ латекс]

Фазовые углы

Однако нужно иметь в виду, что здесь напряжение и ток не совпадают по фазе. Как было показано ранее, напряжение имеет фазовый сдвиг + 90 ° по отношению к току.Если мы представим эти фазовые углы напряжения и тока математически в виде комплексных чисел, мы обнаружим, что сопротивление катушки индуктивности току также имеет фазовый угол:

[латекс] \ text {Opposition} = \ frac {\ text {Voltage}} {\ text {Current}} [/ latex]

[латекс] \ text {Opposition} = \ frac {10 V \ angle \ text {90 °}} {2.6526A \ angle \ text {90 °}} [/ латекс]

[латекс] \ begin {align} \ text {Opposition} = & 3.7699 \ Omega \ angle \ text {90 °} \\ \ text {или} & 0 + j3.7699 \ Omega \ end {align} [/ latex]

Математически мы говорим, что фазовый угол сопротивления катушки индуктивности току равен 90 °, что означает, что сопротивление катушки индуктивности току является положительной мнимой величиной. Этот фазовый угол реактивного противодействия току становится критически важным при анализе цепей, особенно для сложных цепей переменного тока, где реактивное сопротивление и сопротивление взаимодействуют. Будет полезно представить любого компонента против тока в терминах комплексных чисел, а не скалярных величин сопротивления и реактивного сопротивления.

  • Индуктивное реактивное сопротивление — это противодействие, которое индуктор предлагает переменному току из-за сдвинутого по фазе накопления и высвобождения энергии в его магнитном поле. Реактивное сопротивление обозначается заглавной буквой «X» и измеряется в омах, как и сопротивление (R).
  • Индуктивное реактивное сопротивление можно рассчитать по следующей формуле: X L = 2πfL
  • Угловая скорость цепи переменного тока — это еще один способ выразить ее частоту в единицах электрических радиан в секунду вместо циклов в секунду.Его символизирует строчная греческая буква «омега» или ω.
  • Индуктивное реактивное сопротивление увеличивается с увеличением частоты. Другими словами, чем выше частота, тем сильнее она противодействует потоку электронов переменного тока.

Возьмем эту схему в качестве примера для работы:

Последовательная цепь индуктивности резистора: ток отстает от приложенного напряжения от 0 ° до 90 °.

Резистор будет обеспечивать сопротивление 5 Ом переменному току независимо от частоты, а катушка индуктивности — 3.Реактивное сопротивление 7699 Ом на переменный ток при 60 Гц.

Поскольку сопротивление резистора является действительным числом (5 Ом 0 ° или 5 + j0 Ом), а реактивное сопротивление катушки индуктивности — мнимым числом (3,7699 Ом ∠ 90 ° или 0 + j3,7699 Ом), комбинированный эффект двух компонентов будет противодействовать току, равному комплексной сумме двух чисел.

Это комбинированное противодействие будет векторной комбинацией сопротивления и реактивного сопротивления. Чтобы выразить эту оппозицию лаконично, нам нужен более полный термин для обозначения сопротивления току, чем просто сопротивление или реактивное сопротивление.

Этот термин называется импедансом , его символ — Z, и он также выражается в омах, как сопротивление и реактивное сопротивление. В приведенном выше примере полное сопротивление цепи составляет:

Сопротивление по закону Ома

Импеданс связан с напряжением и током, как и следовало ожидать, аналогично сопротивлению в законе Ома:

На самом деле, это гораздо более полная форма закона Ома, чем то, чему учили в электронике постоянного тока (E = IR), так же как импеданс является гораздо более полным выражением сопротивления потоку тока, чем сопротивление. Любое сопротивление и любое реактивное сопротивление, отдельно или в комбинации (последовательно / параллельно), могут и должны быть представлены как единый импеданс в цепи переменного тока.

Чтобы рассчитать ток в приведенной выше схеме, нам сначала нужно задать опорный фазовый угол для источника напряжения, который обычно считается равным нулю. (Фазовые углы резистивного и индуктивного импеданса равны , всегда 0 ° и + 90 °, соответственно, независимо от заданных фазовых углов для напряжения или тока).

Как и в чисто индуктивной схеме, волна тока отстает от волны напряжения (источника), хотя на этот раз отставание не так велико: всего 37,016 ° вместо полных 90 °, как в случае чисто индуктивного схема.

Ток отстает от напряжения в последовательной цепи L-R.

Для резистора и катушки индуктивности соотношение фаз между напряжением и током не изменилось.Напряжение на резисторе синфазно (сдвиг 0 °) с током через него, а напряжение на катушке индуктивности на + 90 ° не совпадает по фазе с током, проходящим через него. Мы можем проверить это математически:

Напряжение на резисторе имеет тот же фазовый угол, что и ток через него, что говорит нам о том, что E и I находятся в фазе (только для резистора).

Напряжение на катушке индуктивности имеет фазовый угол 52.984 °, в то время как ток через катушку индуктивности имеет фазовый угол -37,016 °, разница между ними составляет ровно 90 °. Это говорит нам о том, что E и I все еще не совпадают по фазе на 90 ° (только для катушки индуктивности).

Используйте закон Кирхгофа о напряжении

Мы также можем математически доказать, что эти комплексные значения в сумме составляют общее напряжение, как и предсказывает закон Кирхгофа:

Давайте возьмем те же компоненты для нашей схемы последовательного примера и подключим их параллельно:

Рисунок 6.7 Параллельная цепь R-L.

Поскольку источник питания имеет ту же частоту, что и схема последовательного примера, а резистор и катушка индуктивности имеют одинаковые значения сопротивления и индуктивности, соответственно, они также должны иметь одинаковые значения импеданса. Итак, мы можем начать нашу таблицу анализа с тех же «заданных» значений:

Таблица 6.2 Таблица анализа импеданса 1

Единственная разница в нашей методике анализа на этот раз состоит в том, что мы будем применять правила для параллельных цепей вместо правил для последовательных цепей.Принцип такой же, как и для DC. Мы знаем, что напряжение распределяется равномерно между всеми компонентами в параллельной цепи, поэтому мы можем передать значение общего напряжения (10 вольт 0 °) на все столбцы компонентов:

Таблица 6.3. Таблица анализа импеданса 2

Теперь мы можем применить закон Ома (I = E / Z) по вертикали к двум столбцам таблицы, рассчитав ток через резистор и ток через катушку индуктивности:

Таблица 6.4. Таблица анализа импеданса 3

Так же, как и в случае цепей постоянного тока, токи ответвления в параллельной цепи переменного тока складываются в общий ток (Закон Кирхгофа по току все еще сохраняется для переменного тока, как и для постоянного тока):

Таблица 6.5 Таблица анализа импеданса 4

Наконец, общий импеданс можно рассчитать с помощью закона Ома (Z = E / I) по вертикали в столбце «Всего». Между прочим, параллельный импеданс также можно рассчитать, используя обратную формулу, идентичную той, которая используется при вычислении параллельных сопротивлений.

[латекс] \ tag {6.1} Z_ {parallel} = \ frac {1} {\ frac {1} {Z_1} + \ frac {1} {Z_2} + \ dots \ frac {1} {Z_n}} [ / латекс]

Единственная проблема с использованием этой формулы заключается в том, что она обычно требует большого количества нажатий клавиш калькулятора.И если вы полны решимости использовать такую ​​формулу, как эта «от руки», будьте готовы к очень большому объему работы! Но, как и в случае с цепями постоянного тока, у нас часто есть несколько вариантов расчета величин в наших таблицах анализа, и этот пример ничем не отличается. Независимо от того, каким способом вы рассчитываете полное сопротивление (закон Ома или обратная формула), вы получите одно и то же число:

Таблица 6.6 Таблица анализа импеданса 5

  • Импедансом (Z) управляют так же, как и сопротивлением (R) при анализе параллельной цепи: параллельные импедансы уменьшаются, чтобы сформировать общий импеданс, используя обратную формулу.Только обязательно выполняйте все вычисления в сложной (не скалярной) форме!

[латекс] Z_ {parallel} = \ frac {1} {(\ frac {1} {Z1} + \ frac {1} {Z2} +… \ Frac {1} {Zn})} [/ латекс]

  • Закон Ома для цепей переменного тока:

[латекс] E = {I} {Z} [/ латекс]; [латекс] I = \ frac {E} {Z} [/ latex]; [латекс] Z = \ frac {E} {I} [/ latex]

  • Когда резисторы и катушки индуктивности смешаны вместе в параллельных цепях (так же, как в последовательных цепях), общий импеданс будет иметь фазовый угол где-то между 0 ° и + 90 °.Ток в цепи будет иметь фазовый угол от 0 ° до -90 °.
  • Параллельные цепи переменного тока обладают теми же фундаментальными свойствами, что и параллельные цепи постоянного тока: напряжение равномерно по всей цепи, токи ответвления складываются, образуя общий ток, а импедансы уменьшаются (по обратной формуле), образуя общий импеданс.

В идеальном случае индуктор действует как чисто реактивное устройство. То есть его противодействие переменному току строго основано на индуктивной реакции на изменения тока, а не на трении электронов, как в случае с резистивными компонентами.Однако индукторы не так чисты в своем реактивном поведении. Начнем с того, что они сделаны из проволоки, и мы знаем, что все проволоки обладают некоторой измеримой величиной сопротивления (кроме сверхпроводящей проволоки). Это встроенное сопротивление действует так, как если бы оно было подключено последовательно с идеальной индуктивностью катушки, например:

Рисунок 6.8 Катушка индуктивности Эквивалентная схема реальной катушки индуктивности.

Следовательно, полное сопротивление любой реальной катушки индуктивности всегда будет представлять собой сложную комбинацию сопротивления и индуктивного реактивного сопротивления.

Эту проблему усугубляет то, что называется скин-эффектом , который представляет собой тенденцию переменного тока проходить через внешние области поперечного сечения проводника, а не через середину. Когда электроны движутся в одном направлении (постоянный ток), они используют для движения всю площадь поперечного сечения проводника. С другой стороны, электроны, меняющие направление потока, стремятся избежать прохождения через самую середину проводника, ограничивая доступную эффективную площадь поперечного сечения. Скин-эффект становится более выраженным с увеличением частоты.

Кроме того, переменное магнитное поле индуктора, питаемого переменным током, может излучаться в космос как часть электромагнитной волны, особенно если переменный ток имеет высокую частоту. Эта излучаемая энергия не возвращается к катушке индуктивности и поэтому проявляется в виде сопротивления (рассеяния мощности) в цепи.

Помимо резистивных потерь в проводе и излучения, в индукторах с железным сердечником действуют и другие эффекты, которые проявляются как дополнительное сопротивление между выводами.Когда на индуктор подается переменный ток, создаваемые переменные магнитные поля имеют тенденцию индуцировать циркулирующие токи внутри железного сердечника, известные как вихревые токи , . Эти электрические токи в железном сердечнике должны преодолевать электрическое сопротивление, обеспечиваемое железом, который не так хорош в качестве проводника, как медь. Вихретоковым потерям в первую очередь противодействуют, разделив железный сердечник на множество тонких листов (пластин), каждый из которых отделен от другого тонким слоем электроизоляционного лака.Поскольку поперечное сечение сердечника разделено на множество электрически изолированных участков, ток не может циркулировать в пределах этой площади поперечного сечения, и из-за этого не будет (или будет очень мало) резистивных потерь.

Как и следовало ожидать, потери на вихревые токи в металлических сердечниках индуктора проявляются в виде тепла. Эффект более выражен на более высоких частотах и ​​может быть настолько сильным, что иногда его используют в производственных процессах для нагрева металлических предметов! Фактически, этот процесс «индукционного нагрева» часто используется при литье металлов высокой чистоты, где металлические элементы и сплавы должны нагреваться в вакууме, чтобы избежать загрязнения воздухом, и, следовательно, там, где стандартная технология нагрева сжиганием была бы бесполезной.Это «бесконтактная» технология, когда нагретое вещество не должно касаться катушки (катушек), создающей магнитное поле.

В высокочастотной среде вихревые токи могут возникать даже в поперечном сечении самого провода, способствуя возникновению дополнительных резистивных эффектов. Чтобы противодействовать этой тенденции, можно использовать специальный провод, сделанный из очень тонких, индивидуально изолированных жил, который называется Litz wire (сокращенно от Litzendraht ). Изоляция, отделяющая жилы друг от друга, предотвращает циркуляцию вихревых токов по всей площади поперечного сечения провода.

Кроме того, любой магнитный гистерезис, который необходимо преодолевать при каждом изменении направления магнитного поля индуктора, представляет собой расход энергии, который проявляется как сопротивление в цепи. Некоторые материалы сердечника (например, феррит) особенно известны своим гистерезисным эффектом. Противодействовать этому эффекту лучше всего путем правильного выбора материала сердечника и ограничения пиковой напряженности магнитного поля, генерируемой в каждом цикле.

В целом паразитные резистивные свойства реального индуктора (сопротивление провода, радиационные потери, вихревые токи и гистерезисные потери) выражаются одним термином «эффективное сопротивление»:

Рисунок 6.9 Эквивалентная схема реального индуктора со скин-эффектом, излучением, вихревыми токами и гистерезисными потерями.

Следует отметить, что скин-эффект и потери на излучение применимы к прямым отрезкам провода в цепи переменного тока так же хорошо, как и к спиральному проводу. Обычно их совокупный эффект слишком мал, чтобы его можно было заметить, но на радиочастотах они могут быть довольно большими. Например, антенна радиопередатчика спроектирована специально для рассеивания наибольшего количества энергии в виде электромагнитного излучения.

Конденсаторы Vs. Резисторы

Конденсаторы ведут себя иначе, чем резисторы. В то время как резисторы пропускают через себя поток электронов, прямо пропорциональный падению напряжения, конденсаторы противодействуют изменениям напряжения, потребляя или подавая ток, когда они заряжаются или разряжаются до нового уровня напряжения. Поток электронов «через» конденсатор прямо пропорционален скорости изменения напряжения на конденсаторе. Это противодействие изменению напряжения является еще одной формой реактивного сопротивления , но оно прямо противоположно тому, которое демонстрируют индукторы.

Характеристики цепи конденсатора

Выражаясь математически, соотношение между током, протекающим через конденсатор, и скоростью изменения напряжения на конденсаторе как таковое:

[латекс] i = C \ frac {d_e} {d_t} [/ латекс]

Выражение de / dt получено из расчетов, означающее скорость изменения мгновенного напряжения (e) во времени в вольтах в секунду. Емкость (C) выражается в фарадах, а мгновенный ток (i), конечно, выражается в амперах.Иногда скорость мгновенного изменения напряжения с течением времени выражается как dv / dt вместо de / dt: вместо напряжения используется строчная буква «v» или «e», но это означает то же самое. Чтобы показать, что происходит с переменным током, давайте проанализируем простую схему конденсатора:

Рисунок 6.10 Чистая емкостная цепь: напряжение конденсатора отстает от тока конденсатора на 90 °

Если бы мы изобразили ток и напряжение для этой очень простой схемы, это выглядело бы примерно так:

Рисунок 6.11 Формы сигналов чисто емкостной цепи.

Помните, что ток через конденсатор — это реакция на изменение напряжения на нем на . Следовательно, мгновенный ток равен нулю всякий раз, когда мгновенное напряжение находится на пике (нулевое изменение или наклон уровня на синусоидальной волне напряжения), а мгновенный ток находится на пике везде, где мгновенное напряжение имеет максимальное изменение (точки самый крутой наклон на волне напряжения, где она пересекает нулевую линию).Это приводит к появлению волны напряжения, которая на -90 ° не совпадает по фазе с волной тока. Глядя на график, кажется, что волна тока имеет «фору» по сравнению с волной напряжения; ток «опережает» напряжение, а напряжение «отстает» от тока.

Рисунок 6.12 Напряжение отстает от тока на 90 ° в чисто емкостной цепи.

Как вы могли догадаться, та же необычная волна мощности, которую мы видели в простой цепи индуктивности, присутствует и в простой цепи конденсатора:

Рисунок 6.13 В чисто емкостной цепи мгновенная мощность может быть положительной или отрицательной.

Как и в случае с простой схемой индуктивности, сдвиг фазы на 90 градусов между напряжением и током приводит к появлению волны мощности, которая в равной степени чередуется между положительной и отрицательной полярностью. Это означает, что конденсатор не рассеивает мощность, поскольку он реагирует на изменения напряжения; он просто поочередно поглощает и высвобождает энергию.

Реактивное сопротивление конденсатора

Противодействие конденсатора изменению напряжения означает сопротивление переменному напряжению в целом, которое по определению всегда изменяется по мгновенной величине и направлению.Для любой данной величины переменного напряжения на данной частоте конденсатор данного размера будет «проводить» определенную величину переменного тока. Так же, как ток через резистор является функцией напряжения на резисторе и сопротивления, предлагаемого резистором, переменный ток через конденсатор является функцией переменного напряжения на нем и реактивного сопротивления , обеспечиваемого конденсатором. Как и в случае катушек индуктивности, реактивное сопротивление конденсатора выражается в омах и обозначается буквой X (или, если точнее, X C ).

Поскольку конденсаторы «проводят» ток пропорционально скорости изменения напряжения, они будут пропускать больше тока при более быстром изменении напряжения (поскольку они заряжаются и разряжаются до тех же пиков напряжения за меньшее время) и меньший ток при более медленном изменении напряжения. . Это означает, что реактивное сопротивление в Ом для любого конденсатора равно , обратно пропорционально частоте переменного тока.

[латекс] X_C = \ frac {1} {2πfC} [/ латекс]

Реактивное сопротивление конденсатора 100 мкФ:
Частота (Герцы) Реактивное сопротивление (Ом)
60 26.5258
120 13,2629
2500 0,6366

Обратите внимание, что отношение емкостного реактивного сопротивления к частоте прямо противоположно отношению индуктивного реактивного сопротивления. Емкостное реактивное сопротивление (в омах) уменьшается с увеличением частоты переменного тока. И наоборот, индуктивное реактивное сопротивление (в омах) увеличивается с увеличением частоты переменного тока. Катушки индуктивности противодействуют более быстрому изменению токов, создавая большие падения напряжения; Конденсаторы противодействуют более быстрому изменению падений напряжения, допуская большие токи.

Как и в случае с индукторами, член 2πf в уравнении реактивного сопротивления может быть заменен строчной греческой буквой Омега (ω), которая обозначается как угловая скорость цепи переменного тока. Таким образом, уравнение X C = 1 / (2πfC) также может быть записано как X C = 1 / (ωC), где ω приведено в единицах радиан в секунду .

Переменный ток в простой емкостной цепи равен напряжению (в вольтах), деленному на емкостное реактивное сопротивление (в омах), точно так же, как переменный или постоянный ток в простой резистивной цепи равен напряжению (в вольтах), деленному на сопротивление (в Ом).Следующая схема иллюстрирует это математическое соотношение на примере:

Емкостное реактивное сопротивление.

[латекс] X_C = 26,5258 Ом [/ латекс]

[латекс] I = \ frac {E} {X} [/ латекс]

[латекс] I = \ frac {10} {26,5258 Ом} [/ латекс]

[латекс] I = 0,3770A [/ латекс]

Однако нужно иметь в виду, что здесь напряжение и ток не совпадают по фазе. Как было показано ранее, ток имеет фазовый сдвиг + 90 ° по отношению к напряжению.Если мы математически представим эти фазовые углы напряжения и тока, мы сможем вычислить фазовый угол реактивного сопротивления конденсатора току.

Напряжение в конденсаторе отстает от тока на 90 °.

Математически мы говорим, что фазовый угол сопротивления конденсатора току составляет -90 °, что означает, что сопротивление конденсатора току является отрицательной мнимой величиной. (См. Рисунок выше). Этот фазовый угол реактивного противодействия току становится критически важным при анализе цепей, особенно для сложных цепей переменного тока, где реактивное сопротивление и сопротивление взаимодействуют.Будет полезно представить любого компонента против тока в терминах комплексных чисел, а не только скалярных величин сопротивления и реактивного сопротивления.

  • Емкостное реактивное сопротивление — это противодействие, которое конденсатор предлагает переменному току из-за его сдвинутого по фазе накопления и выделения энергии в его электрическом поле. Реактивное сопротивление обозначается заглавной буквой «X» и измеряется в омах, как и сопротивление (R).
  • Емкостное реактивное сопротивление можно рассчитать по следующей формуле: XC = 1 / (2πfC)
  • Емкостное реактивное сопротивление уменьшается с увеличением частоты.Другими словами, чем выше частота, тем меньше он противодействует (тем больше «проводит») переменному току.

Используя компоненты того же значения в нашей схеме последовательного примера, мы подключим их параллельно и посмотрим, что произойдет:

Рисунок 6.14 Параллельная цепь RC.

Параллельный резистор и конденсатор

Поскольку источник питания имеет ту же частоту, что и схема последовательного примера, а резистор и конденсатор имеют одинаковые значения сопротивления и емкости, соответственно, они также должны иметь одинаковые значения импеданса.Итак, мы можем начать нашу таблицу анализа с тех же «заданных» значений:

Таблица 6.7

Теперь это параллельная схема, и мы знаем, что напряжение распределяется поровну между всеми компонентами, поэтому мы можем поместить цифру для общего напряжения (10 вольт 0 °) во все столбцы:

Таблица 6.8 Расчет

с использованием закона Ома

Теперь мы можем применить закон Ома (I = E / Z) по вертикали к двум столбцам в таблице, рассчитав ток через резистор и ток через конденсатор:

Таблица 6.9

Так же, как и в цепях постоянного тока, токи ответвления в параллельной цепи переменного тока складываются в общий ток (снова Закон Кирхгофа):

Таблица 6.10

Наконец, общий импеданс можно рассчитать с помощью закона Ома (Z = E / I) по вертикали в столбце «Всего». Как мы видели в главе об индуктивности переменного тока, параллельный импеданс также можно рассчитать, используя обратную формулу, идентичную той, которая используется при вычислении параллельных сопротивлений. Следует отметить, что это правило параллельного импеданса остается в силе независимо от типа импедансов, подключенных параллельно.Другими словами, не имеет значения, рассчитываем ли мы схему, состоящую из параллельных резисторов, параллельных катушек индуктивности, параллельных конденсаторов или какой-либо их комбинации: в форме импедансов (Z) все термины являются общими и могут применяться равномерно по той же формуле. И снова формула параллельного импеданса выглядит так:

[латекс] Z_ {parallel} = \ frac {1} {\ frac {1} {Z_1} + \ frac {1} {Z_2} + \ dots \ frac {1} {Z_n}} [/ латекс]

Единственным недостатком использования этого уравнения является значительный объем работы, необходимой для его вычисления, особенно без помощи калькулятора, способного манипулировать сложными величинами.Независимо от того, как мы рассчитываем полное сопротивление для нашей параллельной цепи (закон Ома или обратная формула), мы получим ту же цифру:

  • Импедансом (Z) управляют так же, как и сопротивлением (R) при анализе параллельной цепи: параллельные импедансы уменьшаются, чтобы сформировать общий импеданс, используя обратную формулу. Только не забудьте
    • выполнять все вычисления в комплексном (не скалярном) виде! ZTotal = 1 / (1 / Z1 + 1 / Z2 +.. 1 / Zn)
    • Закон Ома для цепей переменного тока: E = IZ; I = E / Z; Z = E / I
    • Когда резисторы и конденсаторы смешиваются вместе в параллельных цепях (так же, как в последовательных цепях), общий импеданс будет иметь фазовый угол где-то между 0 ° и -90 °.Ток в цепи будет иметь фазовый угол от 0 ° до + 90 °.
    • Параллельные цепи переменного тока обладают теми же фундаментальными свойствами, что и параллельные цепи постоянного тока: напряжение равномерно по всей цепи, токи ответвления складываются, образуя общий ток, а импедансы уменьшаются (по обратной формуле), образуя общий импеданс.

(Следующий раздел был адаптирован из: Уроки электрических цепей, Том II, Глава 5 — Реактивное сопротивление и импеданс — R, L и C)

Прежде чем мы начнем исследовать влияние резисторов, катушек индуктивности и конденсаторов, соединенных вместе в одних и тех же цепях переменного тока, давайте кратко рассмотрим некоторые основные термины и факты.

Сопротивление

Это по сути трение против потока тока. В той или иной степени он присутствует во всех проводниках (кроме проводов super !), Особенно в резисторах. Когда переменный ток проходит через сопротивление, возникает падение напряжения, синфазное с током. Сопротивление математически обозначается буквой «R» и измеряется в омах (Ом).

Реактивное сопротивление

Это по существу инерция против тока.Он присутствует везде, где электрические или магнитные поля развиваются пропорционально приложенному напряжению или току, соответственно; но особенно в конденсаторах и катушках индуктивности. Когда переменный ток проходит через чистое реактивное сопротивление, возникает падение напряжения, которое на 90 ° не совпадает по фазе с током. Реактивное сопротивление математически обозначается буквой «X» и измеряется в омах (Ом).

Импеданс

Это исчерпывающее выражение любых форм противодействия протеканию тока, включая как сопротивление, так и реактивное сопротивление.Он присутствует во всех схемах и во всех компонентах. Когда переменный ток проходит через полное сопротивление, возникает падение напряжения, которое находится где-то между 0 ° и 90 ° не в фазе с током. Импеданс математически обозначается буквой «Z» и измеряется в единицах Ом (Ом) в сложной форме.

Идеальные резисторы обладают сопротивлением, но не реактивным сопротивлением. Идеальные катушки индуктивности и идеальные конденсаторы обладают реактивным сопротивлением, но не имеют сопротивления. Все компоненты обладают импедансом, и из-за этого универсального качества имеет смысл перевести все значения компонентов (сопротивление, индуктивность, емкость) в общие термины импеданса в качестве первого шага при анализе цепи переменного тока.

Рис. 6.15. Идеальный резистор, катушка индуктивности и конденсатор.

Фазовый угол импеданса для любого компонента — это фазовый сдвиг между напряжением на этом компоненте и током через этот компонент. Для идеального резистора падение напряжения и ток всегда находятся в фазе , поэтому угол импеданса резистора считается равным 0 °. Для идеального индуктора падение напряжения всегда опережает ток на 90 °, поэтому фазовый угол импеданса индуктора равен + 90 °.Для идеального конденсатора падение напряжения всегда отстает от тока на 90 °, поэтому считается, что фазовый угол импеданса конденсатора составляет -90 °.

Полное сопротивление переменного тока ведет себя аналогично сопротивлению в цепях постоянного тока: они добавляются последовательно и уменьшаются параллельно. Пересмотренная версия закона Ома, основанная на импедансе, а не на сопротивлении, выглядит так:

Закон Ома для цепи переменного тока

[латекс] \ begin {align} \ tag {6.2} \ text {E} & = {I} {Z} \\ \ text {I} & = \ frac {E} {Z} \\ \ text {Z } & = \ frac {E} {I} \ end {align} [/ latex]

Мы можем взять те же компоненты из последовательной схемы и переставить их в параллельную конфигурацию для простого примера схемы:

Рисунок 6.16 Пример параллельной схемы R, L и C.

Импеданс в параллельных компонентах

Тот факт, что эти компоненты подключены параллельно, а не последовательно, теперь абсолютно не влияет на их индивидуальные импедансы. Пока источник питания имеет ту же частоту, что и раньше, индуктивное и емкостное сопротивление вообще не изменится.

Рисунок 6.17 Пример параллельной цепи R, L и C с импедансами, заменяющими значения компонентов.

Со всеми значениями компонентов, выраженными как импедансы (Z), мы можем создать таблицу анализа и действовать, как в предыдущем примере задачи, за исключением того, что на этот раз следуя правилам параллельных цепей вместо последовательного.

Зная, что напряжение распределяется поровну между всеми компонентами в параллельной цепи, мы можем перенести цифру для общего напряжения во все столбцы компонентов в таблице:

Значения компонентов в таблице 6.11 выражаются в виде импеданса изображения 2

Теперь мы можем применить закон Ома (I = E / Z) по вертикали в каждом столбце, чтобы определить ток через каждый компонент:

Таблица 6.12. Значения компонентов выражаются как изображение импеданса 3

Расчет полного тока и полного импеданса

Существует две стратегии расчета полного тока и полного сопротивления.Во-первых, мы могли рассчитать общий импеданс из всех отдельных параллельных сопротивлений (Z Total = 1 / (1 / Z R + 1 / Z L + 1 / Z C ), а затем вычислить общий ток путем деления напряжения источника на полное сопротивление (I = E / Z).

Тем не менее, работа с уравнением параллельного импеданса с комплексными числами — непростая задача, учитывая все взаимные колебания (1 / Z). Это особенно верно, если вам не повезло, что у вас нет калькулятора, который обрабатывает комплексные числа, и вы вынуждены делать все это вручную (преобразовать индивидуальные импедансы в полярную форму, затем преобразовать их все в прямоугольную форму для сложения, а затем преобразовать обратно в полярную форму для окончательной инверсии, затем инвертировать).Второй способ рассчитать общий ток и полное сопротивление — сложить все токи ответвления, чтобы получить общий ток (полный ток в параллельной цепи — переменный или постоянный — равен сумме токов ответвления), а затем использовать закон Ома. для определения полного сопротивления по общему напряжению и общему току (Z = E / I).

Таблица 6.13 Расчет полного тока и полного импеданса

Любой из методов, выполненный должным образом, даст правильные ответы.

За заметным исключением расчетов мощности (P), все расчеты цепей переменного тока основаны на тех же общих принципах, что и расчеты для цепей постоянного тока.Единственное существенное отличие состоит в том, что в расчетах переменного тока используются комплексные величины, в то время как в расчетах постоянного тока используются скалярные величины. Закон Ома, законы Кирхгофа и даже сетевые теоремы, изученные на постоянном токе, по-прежнему верны для переменного тока, когда напряжение, ток и импеданс выражаются комплексными числами. Те же стратегии поиска и устранения неисправностей, которые применяются в цепях постоянного тока, справедливы и для переменного тока, хотя с переменным током, безусловно, труднее работать из-за фазовых углов, которые не регистрируются портативным мультиметром.

Power — это отдельная тема, которая будет рассмотрена в отдельной главе этой книги. Поскольку мощность в реактивной цепи одновременно поглощается и высвобождается, а не просто рассеивается, как в случае с резисторами, ее математическая обработка требует более прямого применения тригонометрии для решения.

При анализе цепи переменного тока первым шагом в анализе является преобразование всех значений компонентов резистора, катушки индуктивности и конденсатора в импедансы (Z) в зависимости от частоты источника питания.После этого выполните те же шаги и стратегии, которые были изучены для анализа цепей постоянного тока, используя новую форму закона Ома: E = IZ; I = E / Z; и Z = E / I

Помните, что только рассчитанные значения, выраженные в полярной форме , применимы непосредственно к эмпирическим измерениям напряжения и тока. Прямоугольные обозначения — это просто полезный инструмент для сложения и вычитания сложных величин. Полярная запись, где величина (длина вектора) напрямую связана с величиной измеренного напряжения или тока, а угол напрямую связан с фазовым сдвигом в градусах, является наиболее практичным способом выражения сложных величин для анализа схем.

Что такое чисто резистивная цепь? — Фазорная диаграмма и осциллограмма

Цепь, содержащая только чистое сопротивление R Ом в цепи переменного тока, известна как Чистая резистивная цепь переменного тока . Наличие индуктивности и емкости не существует в чисто резистивной цепи. Переменный ток и напряжение движутся как вперед, так и назад в обоих направлениях цепи. Следовательно, переменный ток и напряжение соответствуют форме синусоидальной волны или известной как синусоидальная форма волны.

Состав:

В чисто резистивной схеме мощность рассеивается резисторами, а фазы напряжения и тока остаются неизменными, то есть напряжение и ток достигают своего максимального значения одновременно. Резистор — это пассивное устройство, которое не производит и не потребляет электроэнергию. Он преобразует электрическую энергию в тепло .

Описание резистивной цепи

В цепи переменного тока отношение напряжения к току зависит от частоты источника питания, угла сдвига фаз и разности фаз.В резистивной цепи переменного тока значение сопротивления резистора будет одинаковым независимо от частоты питания.

Пусть переменное напряжение, приложенное к цепи, определяется уравнением

Тогда мгновенное значение тока, протекающего через резистор, показанное на рисунке ниже, будет:

Значение тока будет максимальным при ωt = 90 ° или sinωt = 1

Подставив значение sinωt в уравнение (2), мы получим


Фазовый угол и форма волны резистивной цепи

Из уравнений (1) и (3) ясно, что нет разницы фаз между приложенным напряжением и током, протекающим через чисто резистивную цепь, т.е.е. фазовый угол между напряжением и током составляет ноль . Следовательно, в цепи переменного тока, содержащей чистое сопротивление, ток находится в фазе с напряжением, как показано на рисунке ниже.

Форма волны и фазовая диаграмма чисто резистивной цепи

Питание в чисто резистивной цепи

Три цвета: красный, синий и розовый, показанные на кривой мощности или на форме волны, обозначают кривую тока, напряжения и мощности соответственно. Из векторной диаграммы видно, что ток и напряжение находятся в фазе друг с другом, что означает, что значение тока и напряжения достигает своего пика в один и тот же момент времени, а кривая мощности всегда положительна для всех значений тока. и напряжение.

Как и в цепи питания постоянного тока, произведение напряжения и тока известно как мощность в цепи. Точно так же мощность такая же и в цепи переменного тока, с той лишь разницей, что в цепи переменного тока учитывается мгновенное значение напряжения и тока.

Следовательно, мгновенная мощность в чисто резистивной цепи определяется уравнением, показанным ниже:

Мгновенная мощность, p = vi

Средняя мощность, потребляемая в цепи за полный цикл, равна
, поскольку клапан cosωt равен нулю.

Итак, подставив значение cosωt в уравнение (4), значение мощности будет равно Где,

  • P — средняя мощность
  • В среднеквадратичное значение — среднеквадратичное значение напряжения питания
  • I r.m.s — среднеквадратичное значение тока

Следовательно, мощность в чисто резистивной цепи определяется выражением:

Напряжение и ток в чисто резистивной цепи находятся в фазе друг с другом, имея без разности фаз с нулевым фазовым углом.Переменная величина достигает своего пикового значения в интервале одного и того же периода времени, т.е. повышение и падение напряжения и тока происходят одновременно.

Цепи переменного тока серии

RLC | Физика

Цели обучения

К концу этого раздела вы сможете:

  • Рассчитайте полное сопротивление, фазовый угол, резонансную частоту, мощность, коэффициент мощности, напряжение и / или ток в последовательной цепи RLC.
  • Нарисуйте принципиальную схему последовательной цепи RLC.
  • Объясните значение резонансной частоты.

Когда один в цепи переменного тока, все катушки индуктивности, конденсаторы и резисторы препятствуют току. Как они себя ведут, когда все три встречаются вместе? Интересно, что их индивидуальные сопротивления в Ом не складываются просто так. Поскольку катушки индуктивности и конденсаторы ведут себя противоположным образом, они частично полностью нейтрализуют влияние друг друга. На рисунке 1 показана последовательная цепь RLC с источником переменного напряжения, поведение которой является предметом этого раздела.Суть анализа цепи RLC — это частотная зависимость X L и X C , а также влияние, которое они оказывают на фазу зависимости напряжения от тока (установлено в предыдущий раздел). Это приводит к частотной зависимости схемы с важными «резонансными» характеристиками, которые лежат в основе многих приложений, таких как радиотюнеры.

Рисунок 1. Последовательная цепь RLC с источником переменного напряжения.

Комбинированный эффект сопротивления R , индуктивного реактивного сопротивления X L и емкостного реактивного сопротивления X C определяется как импеданс , аналог сопротивления в цепи постоянного тока по переменному току. Ток, напряжение и импеданс в цепи RLC связаны версией закона Ома для переменного тока:

[латекс] {I} _ {0} = \ frac {{V} _ {0}} {Z} \ text {или} {I} _ {\ text {rms}} = \ frac {{V} _ {\ text {rms}}} {Z} \\ [/ latex].

Здесь I 0 — пиковый ток, В 0 — пиковое напряжение источника и Z — полное сопротивление цепи. Единицы импеданса — омы, и его влияние на схему такое, как и следовало ожидать: чем больше импеданс, тем меньше ток. Чтобы получить выражение для Z в терминах R , X L и X C , мы теперь рассмотрим, как напряжения на различных компонентах связаны с источником. Напряжение.Эти напряжения обозначены как В R , В L и В C на рисунке 1. Для сохранения заряда ток должен быть одинаковым в каждой части цепи. всегда, так что мы можем сказать, что токи в R , L и C равны и синфазны. Но из предыдущего раздела мы знаем, что напряжение на катушке индуктивности В L опережает ток на одну четверть цикла, напряжение на конденсаторе В C следует за током на единицу. -четвертая часть цикла, и напряжение на резисторе В R точно совпадает по фазе с током.На рисунке 2 показаны эти отношения на одном графике, а также показано общее напряжение в цепи В = В R + В L + В C , где все четыре напряжения — мгновенные значения. Согласно правилу петли Кирхгофа, полное напряжение вокруг цепи В, также является напряжением источника. {2}} \\ [/ latex],

, который представляет собой полное сопротивление цепи переменного тока серии RLC .Для схем без резистора принять R = 0; для тех, у кого нет индуктора, возьмите X L = 0; а для тех, у кого нет конденсатора, возьмите X C = 0.

Рис. 2. На этом графике показаны отношения напряжений в цепи RLC к току. Напряжения на элементах схемы в сумме равняются напряжению источника, которое, как видно, не совпадает по фазе с током.

Пример 1.Расчет импеданса и тока

Последовательная цепь RLC имеет резистор 40,0 Ом, индуктивность 3,00 мГн и конденсатор 5,00 мкФ. (a) Найдите полное сопротивление цепи при 60,0 Гц и 10,0 кГц, отметив, что эти частоты и значения для L и C такие же, как в Примере 1 и Примере 2 из раздела Реактивное, индуктивное и емкостное. (b) Если источник напряжения имеет В среднеквадратичное значение = 120 В, что будет I среднеквадратичное значение на каждой частоте?

Стратегия

Для каждой частоты мы используем [latex] Z = \ sqrt {{R} ^ {2} + \ left ({X} _ {L} — {X} _ {C} \ right) ^ {2}} \ \ [/ latex], чтобы найти импеданс, а затем закон Ома, чтобы найти ток. { 2}} \\ & = & \ sqrt {\ left (40.{2}} \\ & = & 190 \ text {} \ Omega \ text {at} 10.0 \ text {kHz} \ end {array} \\ [/ latex]

Обсуждение для (а)

В обоих случаях результат почти такой же, как и наибольшее значение, а импеданс определенно не является суммой отдельных значений. Понятно, что X L доминирует на высокой частоте, а X C доминирует на низкой частоте.

Решение для (b)

Текущее значение I среднекв.

[латекс] {I} _ {\ text {rms}} = \ frac {{V} _ {\ text {rms}}} {Z} = \ frac {120 \ text {V}} {531 \ text { } \ Omega} = 0.226 \ text {A} \\ [/ latex] при 60,0 Гц

Наконец, на частоте 10,0 кГц находим

[латекс] {I} _ {\ text {rms}} = \ frac {{V} _ {\ text {rms}}} {Z} = \ frac {120 \ text {V}} {190 \ text { } \ Omega} = 0,633 \ text {A} \\ [/ latex] при 10,0 кГц

Обсуждение для (а)

Ток при 60,0 Гц такой же (с точностью до трех цифр), найденный для одного только конденсатора в примере 2 из раздела «Реактивное сопротивление, индуктивность и емкость». Конденсатор преобладает на низкой частоте. Ток на частоте 10,0 кГц лишь незначительно отличается от того, который был обнаружен для одной катушки индуктивности в Примере 1 из раздела «Реактивное сопротивление, индуктивность и емкость».{2}}} \\ [/ latex]

Реактивные сопротивления изменяются в зависимости от частоты: X L большое на высоких частотах и ​​ X C большое на низких частотах, как мы видели в трех предыдущих примерах. На некоторой промежуточной частоте f 0 реактивные сопротивления будут равны и уравновешены, давая Z = R — это минимальное значение для импеданса и максимальное значение для I среднеквадратичное значение результатов .Мы можем получить выражение для f 0 , взяв

X L = X C .

Замена определений X L и X C ,

[латекс] 2 \ pi f_ {0} L = \ frac {1} {2 \ pi f_ {0} C} \\ [/ latex].

Решение этого выражения для f 0 дает

[латекс] {f} _ {0} = \ frac {1} {2 \ pi \ sqrt {LC}} \\ [/ latex],

, где f 0 — резонансная частота цепи серии RLC .Это также собственная частота , , при которой цепь будет колебаться, если не будет управляться источником напряжения. При f 0 влияние катушки индуктивности и конденсатора компенсируется, так что Z = R и I среднеквадратичное значение является максимальным.

Резонанс в цепях переменного тока аналогичен механическому резонансу, где резонанс определяется как вынужденное колебание — в данном случае вызванное источником напряжения — на собственной частоте системы.Приемник в радиоприемнике представляет собой цепь RLC , которая лучше всего колеблется на ее f 0 . Переменный конденсатор часто используется для регулировки f 0 , чтобы получить желаемую частоту и отклонить другие. Фиг.3 представляет собой график зависимости тока от частоты, иллюстрирующий резонансный пик в I среднеквадратичное значение при f 0 . Две кривые относятся к двум разным цепям, которые различаются только величиной сопротивления в них.Пик ниже и шире для цепи с более высоким сопротивлением. Таким образом, цепь с более высоким сопротивлением не так сильно резонирует и, например, не будет такой избирательной в радиоприемнике.

Рис. 3. График зависимости тока от частоты для двух последовательных цепей RLC, различающихся только величиной сопротивления. Оба имеют резонанс при f 0 , но для более высокого сопротивления он ниже и шире. Источник управляющего переменного напряжения имеет фиксированную амплитуду В 0 .

Пример 2. Расчет резонансной частоты и тока

Для той же последовательной цепи RLC , имеющей резистор 40,0 Ом, индуктивность 3,00 мГн и конденсатор 5,00 мкФ: (a) Найдите резонансную частоту. (b) Рассчитайте I среднеквадратичное значение при резонансе, если В среднеквадратичное значение равно 120 В.

Стратегия

Резонансная частота находится с помощью выражения в [latex] {f} _ {0} = \ frac {1} {2 \ pi \ sqrt {LC}} \\ [/ latex].{-6} \ text {F} \ right)}} = 1,30 \ text {кГц} \ end {array} \\ [/ latex]

Обсуждение для (а)

Мы видим, что резонансная частота находится между 60,0 Гц и 10,0 кГц, двумя частотами, выбранными в предыдущих примерах. Этого и следовало ожидать, поскольку конденсатор преобладает на низкой частоте, а катушка индуктивности — на высокой. Их эффекты такие же на этой промежуточной частоте.

Решение для (b)

Ток определяется законом Ома.В резонансе два реактивных сопротивления равны и компенсируются, так что полное сопротивление равно только сопротивлению. Таким образом,

[латекс] {I} _ {\ text {rms}} = \ frac {{V} _ {\ text {rms}}} {Z} = \ frac {120 \ text {V}} {40.0 \ text { } \ Omega} = 3,00 \ text {A} \\ [/ latex].

Обсуждение для (б)

В резонансе ток больше, чем на более высоких и низких частотах, рассмотренных для той же цепи в предыдущем примере.

Питание в цепях переменного тока серии

RLC

Если ток изменяется с частотой в цепи RLC , то мощность, подаваемая на нее, также зависит от частоты.Но средняя мощность — это не просто ток, умноженный на напряжение, как в чисто резистивных цепях. Как видно на рисунке 2, напряжение и ток в цепи RLC не совпадают по фазе. Между напряжением источника В и током I существует фазовый угол ϕ , который можно найти из

.

[латекс] \ cos \ varphi = \ frac {R} {Z} \\ [/ latex]

Например, на резонансной частоте или в чисто резистивной цепи Z = R , так что [latex] \ text {cos} \ varphi = 1 \\ [/ latex].Это означает, что ϕ = 0º и что напряжение и ток синфазны, как и ожидалось для резисторов. На других частотах средняя мощность меньше, чем на резонансе. Причина в том, что напряжение и ток не совпадают по фазе, а также потому, что I среднеквадратичное значение ниже. Тот факт, что напряжение и ток источника не совпадают по фазе, влияет на мощность, подаваемую в цепь. Можно показать, что средняя мощность составляет

[латекс] {P} _ {\ text {ave}} = {I} _ {\ text {rms}} {V} _ {\ text {rms}} \ cos \ varphi \\ [/ latex],

Таким образом, cos ϕ называется коэффициентом мощности , который может находиться в диапазоне от 0 до 1.Например, при разработке эффективного двигателя желательны коэффициенты мощности, близкие к 1. На резонансной частоте cos ϕ = 1.

Пример 3. Расчет коэффициента мощности и мощности

Для той же последовательной цепи RLC , имеющей резистор 40,0 Ом, индуктивность 3,00 мГн, конденсатор 5,00 мкФ и источник напряжения с В действующее значение 120 В: (a) Рассчитайте коэффициент мощности и фазу угол для f = 60,0 Гц. (б) Какая средняя мощность при 50.0 Гц? (c) Найдите среднюю мощность на резонансной частоте цепи.

Стратегия и решение для (а)

Коэффициент мощности при 60,0 Гц находится из

.

[латекс] \ cos \ varphi = \ frac {R} {Z} \\ [/ latex].

Мы знаем, что Z = 531 Ом из Пример 1: Расчет импеданса и тока , так что

[латекс] \ cos \ varphi = \ frac {40.0 \ text {} \ Omega} {531 \ text {} \ Omega} = 0,0753 \ text {at} 60.0 \ text {Hz} \\ [/ latex].

Это небольшое значение указывает на то, что напряжение и ток значительно не совпадают по фазе.{-1} 0,0753 = \ text {85,7º} \ text {at} 60,0 \ text {Hz} \\ [/ latex].

Обсуждение для (а)

Фазовый угол близок к 90 °, что согласуется с тем фактом, что конденсатор доминирует в цепи на этой низкой частоте (чистая цепь RC имеет напряжение и ток, сдвинутые по фазе на 90 °).

Стратегия и решение для (b)

Средняя мощность при 60,0 Гц —

P ср. = I среднеквадратичное значение V среднеквадратичное значение cos ϕ .

I среднеквадратичное значение оказалось равным 0,226 А в . Пример 1: Расчет импеданса и тока . Ввод известных значений дает

P средн. = (0,226 A) (120 В) (0,0753) = 2,04 Вт при 60,0 Гц.

Стратегия и решение для (c)

На резонансной частоте мы знаем, что cos ϕ = 1, и I среднеквадратичное значение оказалось равным 6,00 A в Пример 3: Расчет резонансной частоты и тока .Таким образом, P средн. = (3,00 A) (120 В) (1) = 360 Вт при резонансе (1,30 кГц)

Обсуждение

Как ток, так и коэффициент мощности больше в резонансе, производя значительно большую мощность, чем на высоких и низких частотах.

Мощность, подаваемая в цепь переменного тока серии RLC , рассеивается только за счет сопротивления. Катушка индуктивности и конденсатор имеют входную и выходную энергию, но не рассеивают ее из схемы. Скорее они передают энергию туда и обратно друг другу, а резистор рассеивает именно то, что источник напряжения вводит в цепь.Это предполагает отсутствие значительного электромагнитного излучения от катушки индуктивности и конденсатора, например радиоволн. Такое излучение может происходить и даже быть желательным, как мы увидим в следующей главе об электромагнитном излучении, но оно также может быть подавлено, как в случае в этой главе. Схема аналогична колесу автомобиля, движущегося по гофрированной дороге, как показано на рис. 4. Неровности дороги с равномерным интервалом аналогичны источнику напряжения, приводящему колесо в движение вверх и вниз. Амортизатор аналогичен демпфирующему сопротивлению и ограничивающему амплитуду колебаний.Энергия внутри системы перемещается между кинетической (аналогично максимальному току и энергии, хранящейся в индукторе) и потенциальной энергией, хранящейся в автомобильной пружине (аналогично отсутствию тока и энергии, хранящейся в электрическом поле конденсатора). Амплитуда движения колес максимальна, если неровности дороги встречаются на резонансной частоте.

Рис. 4. Вынужденное, но демпфированное движение колеса на автомобильной пружине аналогично цепи переменного тока серии RLC .Амортизатор гасит движение и рассеивает энергию, аналогично сопротивлению в цепи RLC . Масса и пружина определяют резонансную частоту.

Чистая цепь LC с незначительным сопротивлением колеблется на f 0 , той же резонансной частоте, что и цепь RLC . Он может служить эталоном частоты или схемой часов — например, в цифровых наручных часах. При очень маленьком сопротивлении требуется лишь очень небольшая подводимая энергия для поддержания колебаний.Схема аналогична автомобилю без амортизаторов. Как только он начинает колебаться, он некоторое время продолжает работать на своей собственной частоте. На рисунке 5 показана аналогия между цепью LC и грузом на пружине.

Рис. 5. LC-контур аналогичен массе, колеблющейся на пружине без трения и движущей силы. Энергия движется вперед и назад между катушкой индуктивности и конденсатором, точно так же, как она движется от кинетической к потенциальной в системе масса-пружина.

Исследования PhET: комплект для конструирования цепей (AC + DC), виртуальная лаборатория

Создавайте цепи с конденсаторами, катушками индуктивности, резисторами и источниками переменного или постоянного напряжения и проверяйте их с помощью лабораторных инструментов, таких как вольтметры и амперметры.

Щелкните, чтобы загрузить симуляцию. Запускать на Java.

Сводка раздела

  • Аналогом сопротивления переменного тока является сопротивление Z , комбинированное действие резисторов, катушек индуктивности и конденсаторов, определяемое версией закона Ома для переменного тока:

    [латекс] {I} _ {0} = \ frac {{V} _ {0}} {Z} \ text {или} {I} _ {\ text {rms}} = \ frac {{V} _ {\ text {rms}}} {Z} \\ [/ latex],

    , где I o — пиковый ток, а В o — пиковое напряжение источника.{2}} \\ [/ латекс].

  • Резонансная частота f 0 , при которой X L = X C , составляет

    [латекс] {f} _ {0} = \ frac {1} {2 \ pi \ sqrt {LC}} \\ [/ latex]

  • В цепи переменного тока существует фазовый угол ϕ между напряжением источника В и током I , который можно найти из

    [латекс] \ text {cos} \ varphi = \ frac {R} {Z} \\ [/ latex],

  • ϕ = 0º для чисто резистивной цепи или цепи RLC в резонансе.
  • Средняя мощность, подаваемая в цепь RLC , зависит от фазового угла и определяется выражением

    [латекс] {P} _ {\ text {ave}} = {I} _ {\ text {rms}} {V} _ {\ text {rms}} \ cos \ varphi \\ [/ latex],

    cos ϕ называется коэффициентом мощности, который находится в диапазоне от 0 до 1.

Концептуальные вопросы

1. Зависит ли резонансная частота цепи переменного тока от пикового напряжения источника переменного тока? Объясните, почему да или почему нет.

2. Предположим, у вас есть двигатель с коэффициентом мощности значительно меньше 1.Объясните, почему было бы лучше улучшить коэффициент мощности как метод улучшения выходной мощности двигателя, чем увеличивать входное напряжение.

Задачи и упражнения

1. Цепь RL состоит из резистора 40,0 Ом и катушки индуктивности 3,00 мГн. (a) Найдите его полное сопротивление Z при 60,0 Гц и 10,0 кГц. (b) Сравните эти значения Z со значениями, найденными в Пример 1: Расчет импеданса и тока , в котором также был конденсатор.

2. Схема RC состоит из резистора 40,0 Ом и конденсатора 5,00 мкФ. (а) Найдите его полное сопротивление при 60,0 Гц и 10,0 кГц. (b) Сравните эти значения Z со значениями, найденными в Пример 1: Расчет импеданса и тока , в котором также была катушка индуктивности.

3. Цепь LC состоит из индуктора 3,00 мГн и конденсатора 5,00 мкФ. (а) Найдите его полное сопротивление при 60,0 Гц и 10,0 кГц. (b) Сравните эти значения Z со значениями, найденными в Пример 1: Расчет импеданса и тока , в котором также был резистор.

4. Какова резонансная частота индуктора 0,500 мГн, подключенного к конденсатору 40,0 мкФ?

5. Для приема AM-радио вам нужна цепь RLC , которая может резонировать на любой частоте от 500 до 1650 кГц. Это достигается с помощью фиксированной катушки индуктивности 1,00 мкГн, подключенной к конденсатору переменной емкости. Какой диапазон емкости нужен?

6. Предположим, у вас есть запас индукторов от 1,00 нГн до 10,0Гн и конденсаторов от 1.От 00 пФ до 0,100 F. Каков диапазон резонансных частот, который может быть достигнут при сочетании одной катушки индуктивности и одного конденсатора?

7. Какая емкость необходима для получения резонансной частоты 1,00 ГГц при использовании катушки индуктивности 8,00 нГн?

8. Какая индуктивность необходима для получения резонансной частоты 60,0 Гц при использовании конденсатора 2,00 мкФ?

9. Самая низкая частота в диапазоне FM-радио — 88,0 МГц. (а) Какая индуктивность необходима для создания этой резонансной частоты, если она подключена к 2.Конденсатор 50 пФ? (b) Конденсатор регулируемый, что позволяет регулировать резонансную частоту до 108 МГц. Какой должна быть емкость на этой частоте?

10. Последовательная цепь RLC имеет резистор 2,50 Ом, индуктивность 100 мкГн и конденсатор 80,0 мкФ. (A) Найдите полное сопротивление цепи при 120 Гц. (b) Найдите полное сопротивление цепи на частоте 5,00 кГц. (c) Если источник напряжения имеет В среднеквадратичное значение = 5,60 В, что будет I среднеквадратичное значение на каждой частоте? (d) Какова резонансная частота контура? (e) Что такое I среднеквадратичное значение при резонансе?

11.Последовательная цепь RLC имеет резистор 1,00 кОм, индуктивность 150 мкГн и конденсатор 25,0 нФ. (а) Найдите полное сопротивление цепи при 500 Гц. (b) Найдите полное сопротивление цепи на частоте 7,50 кГц. (c) Если источник напряжения имеет В среднеквадратичное значение = 408 В, что будет I среднеквадратичное значение на каждой частоте? (d) Какова резонансная частота контура? (e) Что такое I среднеквадратичное значение при резонансе?

12. Цепь серии RLC имеет 2.Резистор 50 Ом, катушка индуктивности 100 мкГн и конденсатор 80,0 мкФ. (a) Найдите коэффициент мощности при f = 120 Гц. (б) Каков фазовый угол при 120 Гц? (c) Какая средняя мощность при 120 Гц? (d) Найдите среднюю мощность на резонансной частоте цепи.

13. Последовательная цепь RLC имеет резистор 1,00 кОм, индуктивность 150 мкГн и конденсатор 25,0 нФ. (a) Найдите коэффициент мощности при f = 7,50 Гц. б) Каков фазовый угол на этой частоте? (c) Какая средняя мощность на этой частоте? (d) Найдите среднюю мощность на резонансной частоте цепи. {2}} \\ [/ latex]

резонансная частота:
частота, при которой полное сопротивление в цепи минимально, а также частота, с которой цепь будет колебаться, если не будет управляться источником напряжения; рассчитывается по [latex] {f} _ {0} = \ frac {1} {2 \ pi \ sqrt {\ text {LC}}} \\ [/ latex]
фазовый угол:
обозначается как ϕ , величина, на которую напряжение и ток не совпадают по фазе друг с другом в цепи
Коэффициент мощности:
— величина, на которую мощность, передаваемая в цепи, меньше теоретического максимума цепи из-за того, что напряжение и ток не совпадают по фазе; рассчитывается по cos ϕ

Избранные решения проблем и упражнения

1.(a) 40,02 Ом при 60,0 Гц, 193 Ом при 10,0 кГц (b) При 60 Гц, с конденсатором, Z = 531 Ом, что в 13 раз больше, чем без конденсатора. Конденсатор имеет большое значение на низких частотах. На 10 кГц, с конденсатором Z = 190 Ом, примерно так же, как без конденсатора.

Разное

Добавить комментарий

Ваш адрес email не будет опубликован. Обязательные поля помечены *